MN

Pataasin ang iyong marka sa homework at exams ngayon gamit ang Quizwiz!

During the Victorian era, the United Kingdom's "Informal Empire" consisted of: A.) Protectorates in Africa where the British Crown established military outposts and administrative offices. B.) Dominions in North America and Australia where the British Crown had ceased to function as a colonial ruler. C.) Mandates in the Middle East where the British Crown installed figurehead rulers and governed behind the scenes. D.) Independent nations in Latin America where the British Crown had extensive commercial and financial investments

answer D. The United Kingdom established trade relations with Latin American nations almost as soon as they gained independence from Spain. The relations grew over time until by World War I, the United Kingdom was investing heavily in the financial and transportation infrastructures of countries such as Chile and Argentina. This relationship was called the "Informal Empire" since these lands had never been under British rule. With the onset of World War I however, British interests in the region declined, and American interests swelled

Which of the following historical figures was a German printer credited with the production of a Bible? A. Johann Gutenberg. B. William Shakespeare. C. Miguel de Cervantes. D. Dante Alighieri.

answer is (A). German printer Johann Gutenberg is credited with the production of the Gutenberg Bible.

Which of the following BEST describes the main idea behind social Darwinism? A.) Competition among individuals, groups, nations or ideas drives social evolution. B.) Human beings are not any better than animals on a fundamental level. C.) Substandard members of society should be exterminated. D.) Illness and death among human beings is necessary and good

answer is (A). Social Darwinism draws directly from Charles Darwins' Theory of Natural Selection, in which competition between individual organisms drives biological evolutionary change through survival of the fittest

Which of the following terms describes a group of indigenous peoples inhabiting the Arctic regions of Alaska, Greenland and Canada? A. Inuit. B. Anasazi. C. Iroquois. D. Cherokee.

answer is (A). The Inuit people live throughout most of the Canadian Arctic and subarctic. In Canada and Greenland, the term Eskimo has taken on a pejorative connotation and has been replaced by the term Inuit.

Although it was publicly stated that the purpose of the Philadelphia Convention of 1787 was to revise the Articles of Confederation, the intention of many of the attendees was: A.) To create a new government. B.) To follow through with the public statement. C.) To avoid rocking the political boat. D.) None of the above.

answer is (A). The Philadelphia Convention, also called the Constitutional Convention, resulted in the United States Constitution rather than just a simple revision of the Articles of Confederation

The period of time known as the Roaring 20's is also sometimes called which of the following? A. The Jazz Age. B. The Progressive Era. C. The Music Age. D. The American Revolution.

answer is (A). The Roaring 20's period was also referred to as The Jazz Age due to the upsurgence of modern jazz music

The purpose of the Magna Carta and the English Bill of Rights was to do which of the following? A. To limit the power of the Monarch. B.To allow the practice of any religion. C.To acknowledge the Divine Rights of Kings. D.To expand voting rights within the population

answer is (A). While they did not directly address voting rights or religious freedom, the Magna Carta (1215) and the English Bill of Rights (1689) were both designed with the main intent of transferring some of the Monarch's power to the general populace. The Divine Rights of Kings, or a suggestion of the Monarch's absolute power under God, was precisely the idea against which the Magna Carta and the English Bill of Rights were written

Which of the following proposes that the only way to solve the problems of capitalism is for the working class to replace the ruling class? A. Radicalism. B. Communism. C. Conservativism. D. Libertarianism.

answer is (B). Communism is a socio-economic structure that promotes the establishment of a classless, stateless society based on common ownership of the means of production

Which of the following are the dates of World War I? A. 1889-1900. B. 1914-1918. C. 1920-1925. D. 1939-1945.

answer is (B). Immediately after the assassination of the Archduke Franz Ferdinand in June, 1914, World War I began and ended with several treaties in November, 1918

Jordan, Kuwait and Lebanon are located in which of the following regions? A. North Africa. B. Southwest Asia. C. Sub-Saharan Africa. D. Latin America

answer is (B). Lebanon, Jordan and Kuwait are located in Southwest Asia

Which of the following is TRUE about primary groups? A.) They are usually large in size. B.) They frequently last for long periods of time. C.) They are institutional more often than not. D.) They are formal relationships.

answer is (B). Primary groups consist of small groups with intimate, kin-based relationships that commonly last for years

Which of the following is the MOST significant regarding the Code of Hammurabi? A. It was created entirely by a Mesopotamian king B. It is the best preserved ancient law code. C. It provided guidelines of conduct. D. It detailed basic laws for the common man

answer is (B). The Code of Hammurabi is most significant because it is the only ancient law code that is well-preserved

What was the main purpose of the Sons of Liberty? A.) To educate male children of the 13 colonies about patriotism. B.) To dismantle and disrespect British authority and power. C.) To remind the colonists about the supremacy of men. D.) To encourage loyalty to the Crown

answer is (B). The Sons of Liberty was a secret organization which originated in the 13 colonies during the American Revolution. They attacked both the apparatus and symbols of British authority such as property of the gentry, customs officers, and staged the infamous Boston Tea Party

The President of the United States can serve a maximum of _______ term(s), which consist(s) of ______ years each. A. One term, eight years. B. Two terms, four years. C. One term, two years. D. One term, four years.

answer is (B). The U.S. President may be re-elected one time to serve an additional four-year term, regardless of whether those terms are consecutive.

Although the population of the United States has the right to cast votes for the Presidential election, who actually elects the President? A. The United States Congress. B. The Electoral College. C. The U.S. House of Representatives. D. The U.S. Senate.

answer is (B). The United States has a President who is elected indirectly. Although members of the Electoral College usually cast their votes for the winner of the popular vote in their respective States, they are not always legally required to do so

What was the MAIN goal of President Franklin Roosevelt's New Deal? A.) To ensure that only people from certain classes could hold government jobs. B.) To provide financial relief from the Great Depression. C.) To allow for desegregation of public places. D.) To force the common man to obtain employment.

answer is (B). The purpose of the New Deal was to give relief to the poor, to reform the financial system in the United States, and to recover the economy after the stock market crash

Which of the following is the MOST significant shift in human society from the Paleolithic Period to the Neolithic Period? A. The use of pottery. B. The rise of farming. C. The breeding of animals. D. The polygamous nature of marriage.

answer is (B). The transition from nomadic hunting and gathering communities and bands to agriculture and settlement is the most significant shift between the two Periods and is called the Neolithic Revolution.

The women who lobbied and won passage of the 19th Amendment to the United States Constitution were known as: A. Feminists. B. Suffragettes. C. Liberationists. D. Socialists.

answer is (B). The women who advocated and worked for their own right to vote were known as Suffragettes

In general, the climate of the Caribbean can be described as which of the following? A. Arid. B. Semi-arid. C. Tropical. D. Arctic.

answer is (C). Although the rainfall varies with elevation and water currents, the climate in the Caribbean is tropical.

In which of the following world regions are Belgium, France and Germany located? A. Caribbean. B. North America. C. Western Europe. D. East Asia.

answer is (C). Belgium, France and Germany are located in Western Europe.

Which of the following is BEST defined by the formation of zones of temperature and moisture gradients? A. Precipitation. B. Lapse rate. C. Frontogenesis. D. Coriolis effect.

answer is (C). Frontogenesis, usually the result of uneven solar heating, is the formation of different zones of temperature and moisture gradients

Leviathan, a book that advocated a strong central government to avoid the discord of civil war, was written by which of the following philosophers? A. John Locke. B. Niccolo Machiavelli. C. Thomas Hobbes. D. Vladimir Lenin.

answer is (C). In Leviathan, Hobbes set out his doctrine of the foundation of societies and legitimate governments

Which of the following is best known for his treatises on realist political theory? A. Vladimir Lenin. B. Karl Marx. C. Niccolo Machiavelli. D. Thomas Hobbes.

answer is (C). Machiavelli is best known for his political perspective that leaders must be publicly above reproach, but that they may be required to privately perform immoral acts in order to achieve the end goal

What was the main purpose of the "Reconstruction Amendments"? A. To rebuild the nation after the Civil War. B. To enforce public segregation. C. To expand civil rights for Black Americans. D. To further enforce slavery.

answer is (C). The 13th, 14th and 15th Amendments are considered the "Reconstruction Amendments". Their main purpose was to extend liberty to all people, including slaves and their descendants

Who was President of the United States during the time of The Louisiana Purchase? A. Washington. B. Adams. C. Jefferson. D. Madison.

answer is (C). Thomas Jefferson was President during the time of The Louisiana Purchase

Which of the following are the dates of World War II? A. 1914-1918. B. 1920-1935. C. 1939-1945. D. 1950-1965.

answer is (C). World War II began in September, 1939 with the invasion of Poland by Germany and ended when Japan surrendered in August,1945 after being devastated by atomic bombs in Hiroshima and Nagasaki

Which of the following is NOT a type of headland? A. Peninsula. B. Cape. C. Promontory. D. Sound.

answer is (D). A sound is a type of bay

Which of the following statements is TRUE regarding Alexander the Great? A.) He initiated a democratic form of government. B.) He encouraged debate and scholarly activities. C.) He avoided war at all costs. D.) He united the segregated city-states of Greece into a unified nation.

answer is (D). Alexander the Great transformed the segregated city-states of Greece into a single nation.

Which of the following is BEST defined by the idea that a majority of the population has the right to make most of the decisions that will affect society as a whole? A. Sovereignty. B. Federalism. C. Equality. D. Majoritarianism.

answer is (D). Simply stated, majoritarianism is the ruling of a majority class over a minority class.

The Presidential Election is held during which of the following months? A. January B. March. C. September. D. November.

answer is (D). The Presidential Election is held in November, and the Presidential Inauguration is held in the January that follows

What election is held to fill a mid-term vacancy? A. Special election. B. Regular election. C. Exchange election. D. Election recall.

answer is A. A special election is held between general elections to fill a vacancy that has occurred

"In the new Code of Laws which I suppose it will be necessary for you to make, I desire you would Remember the Ladies, and be more generous and favourable to them than your ancestors. Do not put such unlimited power into the hands of the Husbands. Remember all Men would be tyrants if they could. If particular care and attention is not paid to the Ladies, we are determined to foment a Rebellion, and will not hold ourselves bound by any Laws in which we have no voice, or Representation." Who of the following wrote the preceding passage? A. Abigail Adams. B. Elizabeth Cady Stanton. C. Jane Addams. D. Susan B. Anthony.

answer is A. Abigail Adams, an exponent of women's rights, expressed the quoted sentiments in a letter to her husband, John Adams on March 31, 1776, while the latter was in Philadelphia at the Continental Congress

Asking a stranger for directions is an example of which of the following social interactions? A. Accidental. B. Repeated. C. Regular. D. Regulated.

answer is A. Accidental social interactions are not planned and are unlikely to be repeated, such as asking a stranger for directions

Which of the following was NOT one of the foundations of free market economics according to Adam Smith? A. Charitable contributions. B. Division of labor. C. Pursuit of self-interest. D. Freedom of trade.

answer is A. Adam Smith, in An Inquiry into the Nature and Causes of the Wealth of Nations, espouses three main foundations of free market economics, and charitable giving was not included among said foundations, nor associated with them

The term of office for a member of the United States House of Representatives is: A. Two years. B. Four years. C. Six years. D. Eight years.

answer is A. As Article I, Section 2 of the Constitution states: "The House of Representatives shall be composed of members chosen every second year by the people of the several states, and the Electors in each state shall have the qualifications requisite for Electors of the most numerous branch of the state legislature."

Laws issued directly from the President are called what? A.Executive orders. B. Presidential law. C. De Facto law. D. Executive execution

answer is A. At certain times, the President is allowed to enact certain laws most of which have to do when there is a state of emergency. These automatically become laws although they can be repealed if they are found to be unconstitutional

When interpreting a map, what does 'scale' mean? A.) The ratio of the distance on the map and the corresponding distance on the surface of the earth B.) The difference between the distance on the map and the distance on the globe C.) The fraction of the earth being measured D.) A cartographer's shorthand for a place on the globe

answer is A. Cartographers, or map makers, use scale, the ratio of the distance on the map and the corresponding distance on the surface of the earth, when making maps. Maps are drawn to a reduced scale or they would be as large as the area being represented. A map's scale is expressed as a ratio, such as 1 inch/20,000 miles. Any unit of measurement may be used.

Before Francisco Pizzarro arrived in 1533, the capital of the Inca Empire was: A.Cuzco. B. Machu Picchu. C. Vilcabamba. D. Lima.

answer is A. Cuzco was situated at the intersection of the four regions of the Inca Empire: Chinchasuyu, Antisuyu, Contisuyu, and Collasuyu--and had been the capital since the Empire was established by Manco Capac around 1200 C.E. Cuzco would remain the capital of the Spanish-conquered regions for only two years beyond that time, when Pizarro founded Lima as the capital of the Viceroyalty of Peru

What leader of the Persian forces was the first to try to conquer the Greeks? A. Darius I. B. Xerxes. C. Alexander the Great. D. Darius II.

answer is A. Darius tried to take on the Greeks but under the leadership of Athens he was driven back. Later his son Xerxes would try to take on the Greek forces

The Gaza Strip and West Bank have been fought over by Palestine against what other nation? A. Israel. B. Iraq. C. Jordan. D. Kuwait.

answer is A. Ever since the conception of the state, Israel has had to deal with opposition from the Arabic nations. The West Bank and Gaza Strip are fought over by the Palestinians who believe that land belongs to them.

Which were the first animals domesticated by early people? A. Dogs B. Mammoths C. Sheep D. Cattle

answer is A. Fossilized dog bones have been found alongside human remains dating back 14,000 years. Dogs may have been companions or food. Sheep, pigs, goats and cattle were domesticated in later ages using the same sort of evidence. Mammoths were bigger than elephants and very dangerous. They were hunted by early civilizations

George Washington began his military career fighting for the British. One one of his first commands, he set up "Fort Necessity" in the Ohio River Valley. Why did he set up this fort? A.) He was suprised by a French detachment while on a spying mission. B.) He was commanded to protect the mouth of the Ohio River. C.) He was overseeing the convergence of the two rivers which fed into the Ohio River to keep trading routes open. D.) The fort was needed by the British as a command post in the Ohio River Valley

answer is A. George Washington was a captain in the Virginia militia fighting for the British. He was sent into French territory on an spying mission and ran into a French detachment. The Virginians fired and the French fled. Washington set up a fort called "Fort Necessity." which the French overcame. Luckily for America, they let G.W. and his men go home. The war began over the Ohio River Valley

The 15th Amendment to the Constitution was ratified during the tenure of which of the following Presidents? A. Ulysses S. Grant. B. Andrew Johnson. C. Abraham Lincoln. D. Rutherford B. Hayes.

answer is A. Grant's civil rights record also included signing into law the Civil Rights Act of 1875. A law, however, which became unenforceable due to the complete removal of federal troops from the South following the end of his second term in office.

Which of the following abolitionists was known for her extensive night shifts on the Underground Railroad as well as her scouting exploits for Union troops during the Civil War? A. Harriet Tubman. B. Harriet Beecher Stowe. C. Harriet Ann Jacobs. D. Frances Harriet Whipple.

answer is A. Harriet Tubman was born a slave in Maryland, escaped to Pennsylvania, and spent several subsequent years making daring forays back to the slave-holding South to free her relatives as well as many other slaves. As Frederick Douglass described her: "The midnight sky and the silent stars have been the witnesses of your devotion to freedom and of your heroism."

This general wrote the Art of War. A. Sun-tzu. B. Patton. C. Machiavelli. D. Churchill.

answer is A. He wrote this to help advise the troops on how to act when they were engaged in war. It helps to showcase different tactics for people to use when at war and was greatly admired by Mao Zedong.

When the U.S. soldiers liberated the eastern bloc nations at the end of WWII, what did they discover which horrified them? A. Concentration camps B. Prisoner of war camps C. Training camps for guerrilla warriors D. Hessian mercenaries

answer is A. Hitler's" final solution" resulted in the deaths of 6 million Jews and 4 million non-Jews. The Allied soldiers liberated the "living skeletons" they found in Hitler's concentration camps. The Allies had known about them, but were unprepared for the enormity of the crime.

You live in rural Angola. The law states that you may not cross over the Cunene River to trade in Namibia. You regularly cross the river to trade. What is this an example of? A. Distance decay B. Distance rule C. Degenerative rule D. Degenerative decay

answer is A. If the ruling powers are far enough away that a law can be thwarted with no consequence, it is an example of distance decay. This happened in the U.S.A. during the era we call the "Wild West."

What was the first city to be struck by the atomic bomb? A. Hiroshima. B. Iwo Jima. C. Tokyo. D. Nagisaki.

answer is A. In 1945, the US dropped an atomic bomb to help end the war. The city was almost completely destroyed and today is seen as one of the centers for the movement for worldwide peace.

Workers in which of the following economic sectors organized in the early 20th Century to create industrial unions that included non-skilled laborers? A. Steel and textile mills. B. Newspapers and shoe factories. C. Cigar shops and haberdasheries. D. Construction and railroads.

answer is A. In the 19th Century, the prevailing means of labor organization was along the lines of specific skilled crafts or trades. In the early 20th Century, as the textile and steel industries became more mechanized and employed unskilled workers, a need to include these workers in the process of collective bargaining to gain better working conditions and wages surfaced.

Mahatma Gandhi was seen as a prominent figure in the nationalist movement of what nation? A. India. B. China. C. Sri Lanka. D. Israel.

answer is A. India had been under British rule for years and wanted to get their own freedom. Gandhi introduced the concept of peaceful protest and eventually the Indian people were able to regain their independence.

When the Declaration of Independence was written by Thomas Jefferson in 1776, he listed the grievances of the colonies against the British. Why was this document thought necessary by the members of Congress? A.) It gave the reasons why the colonies had a right to independence. B.) It declared that the British should allow American representation in Parliament. C.) It established the format for the American government. D.) It declared war on the British.

answer is A. Jefferson's document listed the grievances against the British and the right Americans had to declare themselves independent of England. One delegate called it, "Mr. Jefferson's explanation of Mr. Lee's resolution" for independence.

Who took the power of the Soviet Government after the death of Stalin? A. Khrushchev. B. Lenin. C. Romanov. D. Putin.

answer is A. Khrushchev was instrumental in trying to get the Soviet nation to stop depending on the West for their agricultural needs. He was in favor of a strong military which led to a decline in other areas of the economy. This eventually forced him to retire due to unpopularity and a coup

Which of the following became the first President of the first African country to gain independence from Great Britain? A.Kwame Nkrumah. B. Nnamdi Azikiwe. C. Maurice Yaméogo. D. Léopold Senghor.

answer is A. Kwame Nkrumah, a graduate of the University of Pennsylvania, was the leader of Ghana on March 6, 1957, when that country declared its independence from Great Britain. He was subsequently elected Ghana's first President in the democratic elections that were held three years later

What type of government is ruled by religious figures? A. Theocracy. B. Oligarchy. C. Monarchy. D. Republic.

answer is A. Many Middle Eastern nations have been ruled by a religious leader who also made the policies for the nation. This can sometimes lead to conflicts when the law of the land goes against that of the religion.

When the President chooses not to do an action to a bill and allows it to die by running out the time allowed, what is this called? A. Pocket veto. B. Veto. C. Poor politics. D. Dropped bill.

answer is A. Many bills are defeated by the pocket veto because the President chooses not to make a clear stand on the issue. This usually happens if a bill was passed by a legislative body which is different from his own political body. The bill must then go back to the legislative branches where it must now get a larger majority in order to reach the President's desk again.

The NRA (national rifle association) is based upon defending which amendment? A. 2nd. B. 3rd. C. 9th. D. 11th

answer is A. Many people think that the second amendment should be taken down and that guns are leading to the downfall of society. The NRA (National Rifle Association) claims that guns are guaranteed by the government and founding fathers and should not be taken away from every day citizens.

Which of the following BEST describes a significant difference between a recession and a depression? A.) A recession is a normal downturn in the business cycle and a depression is a sustained downturn in the economy. B.) A depression is a normal downturn in the business cycle and a recession is a sustained downturn in the economy. C.) A recession involves excessive currency devaluations and a depression involves reduced amounts of trade and commerce. D.) A depression involves excessive currency devaluations and a recession involves reduced amounts of trade and commerce.

answer is A. One of the most significant differences between a recession and a depression is the severity and length of the economic symptoms. A depression is an exceptionally long and intense period of recession.

What type of government is ruled by the wealthy? A. Plutocracy. B. Theocracy. C. Aristocracy. D. Dictatorship.

answer is A. People who have the most money will be able to influence how the government is going to run. This can lead to an overrun of monopolies against certain elements

Which of the following psychological paradigms specifically addresses repression, sexuality and the unconscious mind? A. Psychoanalysis. B. Behaviorism. C. Existentialism. D. Humanism.

answer is A. Psychoanalysis specifically focuses on subjects that are sometimes considered taboo, such as sexuality and repression

How often are members of the House of Representatives elected? A. 2 years B. 4 years C. 6 years D. Every year

answer is A. Representatives have a smaller amount of time in which they are in office since they are to represent the people of the state on a closer level. Most of the time, people who are elected to these positions may end up in them for longer amounts of time than in other offices

The first English baby born in the New World was born on Roanoke Island. What was her name? A.Virginia Dare B. Elizabeth Dare C. Mary Yost D. Mary Raleigh

answer is A. Roanoke Island was colonized in 1587. Virginia Dare was born to Eleanor Dare, the daughter of the group's leader, John White.The settlement on Roanoke Island vanished in 1590. It is called "The Lost Colony."

The United States Supreme Court case of Chisholm v. Georgia held that under the provisions of Article III, Section 2 of the Constitution, a private citizen could bring suit against a state in a federal court. This decision led states to demand which of the following constitutional amendments? A. 11th. B. 12th. C. 16th. D. 17th.

answer is A. The 11th Amendment, ratified in 1795, reads in full: "The Judicial power of the United States shall not be construed to extend to any suit in law or equity, commenced or prosecuted against one of the United States by citizens of another State, or by citizens or subjects of any foreign state." Futhermore, the Supreme Court case of Ford Motor Co. v. Department of Treasury (1945) states that "the 11th Amendment denies to the federal courts authority to entertain a suit brought by private parties against a State without the State's consent."

Slavery was officially abolished in the United States by the ratification of which of the following? A. Amendment XIII of the Constitution. B. Amendment XIV of the Constitution. C. Civil Rights Act of 1866. D. Emancipation Proclamation.

answer is A. The 13th Amendment was ratified by Congress on December 6, 1865, and states in full as follows: "1. Neither slavery nor involuntary servitude, except as a punishment for crime whereof the party shall have been duly convicted, shall exist within the United States, or any place subject to their jurisdiction. 2. Congress shall have power to enforce this Article by appropriate legislation."

Ben is an immigrant to the U.S. in 1798. He wishes to become a citizen but is told that he has to wait 14 years before he can become naturalized. What happened in 1798 which made this provision? A.) President John Adams signed the Alien and Sedition Acts. B.) The U.S. Constitution was ratified. C.) A slave uprising made the government leery of immigrants. D.) The U.S. and Canada were arguing over their boundaries

answer is A. The Alien and Sedition Act extended the naturalization period from five to fourteen years. It also made it a crime to publish "any false, scandalous and malicious writing" about the president, Congress or the government in general. This act was a dark mark on the freedoms granted by the Constitution

In the 1760s, the British raised taxes for the American colonists which caused much discontent. Why was the reason for this tax increase? A.) The British needed money to pay for the many wars they had fought. B.) The British needed money for expanding into the western territories. C.) The British needed money to pay for expanding industries in London. D.) The British needed money to pay for ships for an impending war with France

answer is A. The British had overextended themselves fighting many wars and needed money. The national debt was equivalent to $120 million dollars. They taxed the colonies to help defray this because the cost of protecting the colonies was part of this debt

A series of events in the 1750s to the 1770s caused changes in British/American relations. Which is not a British imposition which rankled the American colonists? A.) The British wanted to stop the slave trade. B.) The British wanted the colonies to help defray the costs of quartering soldiers. C.) The British wanted to limit westward expansion. D.) The British wanted to stop the smuggling trade

answer is A. The British had overextended themselves fighting many wars and needed money. The national debt was equivalent to $120 million dollars. They taxed the colonies to help defray this because the cost of protecting the colonies was part of this debt. They wanted to stop westward expansion to placate the Native Americans and stop wars there. They wanted to eliminate smugglers who were cutting into their trade. The slave trade continued

We have an old mother that peevish is grown; She snubs us like children that scarce walk alone; She forgets we're grown up and have sense of our own. Ben Franklin 1763 What does the rhyme above refer to? A.) Discontent with British rule in the colonies. B.) A call for independence from England C.) The westward expansion of the colonies D.) The role of the colonies in the British Parliament

answer is A. The British had overextended themselves fighting many wars and needed money. They taxed the colonies to help defray this because the cost of protecting the colonies was part of this debt. They wanted to stop westward expansion to placate the Native Americans and stop wars there. They wanted to eliminate smugglers who were cutting into their trade. The colonists wanted more say in the rules the British Parliament imposed on them. They had no representatives there and no role in deciding their fate. There was not yet a call for independence

The Declaration of Independence lists the rights of a free people. Included are the right to throw off an oppressing government, establish a new one and to pursue life, liberty and happiness. It also stated that nature declares that "all men are created equal." What else was listed in this document? A.) Complaints against the king which justified the break with England B.) A Bill of Rights based on the English Bill of Rights of 1689 C.) A blueprint for a new form of government D.) A plan to reestablish peace with Great Britain

answer is A. The Declaration lists many complaints which justify independence. The Constitution is a "blueprint" for our government. The "Olive Branch" petition asked for peace

The tides of Earth are dependent upon what? A. The Moon. B. The Sun. C. The seasons. D. The day of the week

answer is A. The Moon's gravitational pull on Earth determines when the tides are going to rise or fall during any given day. There are two high tides and two low tides each day

The city of Milan is located along what river in Italy? A. Po. B. Seine. C. Thames. D. Tiber.

answer is A. The Po River was instrumental in early Italian history in transporting goods from the interior of Italy to the Adriatic Sea. Today, it is still one of the major waterways and runs through many of the larger cities in the area

The Civil War had which of the following effects on the Confederate economy? A. Inflation. B. Increased output. C. Increased levels of employment. D. Recovery.

answer is A. The South's output was primarly cotton. However, the North blockaded Southern ports, and prevented cotton from being exported. Because the South had no income, it was forced to print more and more money, resulting in higher and higher inflation

Who leads the House of Representatives in discussions? A. Speaker of the House. B. Vice President. C. President. D. Whip of the House.

answer is A. The Speaker of the House brings up the new issues which the house needs to discuss to find solutions to. This position is usually given to the person from the political party which has the most members in it currently. It changes as more people are brought in

During the term of Thomas Jefferson, the U.S. Supreme court, in a case known as Marbury v. Madison, asserted itself as a branch of government. Marbury v. Madison gave the Supreme Court all except which of the following? A.) It raised the number of justices to nine. B.) It allowed for judicial review. C.) It determined the authority of the court to decide the constitutionality of the acts of Congress or the President. D.) It was the first case in which the court exercised its authority in the case of the constitutionality of acts of Congress or the President.

answer is A. The Supreme Court is the Judicial branch of the government. It was established by the Constitution. Its powers, including the right to rule on elections, are described there. The legislative and executive branches are subject to the decisions the court makes on the constitutionality of their acts. This case was the first in which it did that

The Communist Party of Greece was declared illegal as a result of which of the following? A. The Truman Doctrine. B. The German Occupation. C. The Marshall Plan. D. The Regime of the Colonels.

answer is A. The Truman Doctrine authorized $400 million in economic and military aid to Turkey and Greece in exchange for keeping those two countries free of Communists and Soviet control. As a result, the Communist Party, which had earlier been involved in fighting the Germans during the Occupation, as well as in democratic elections following the Greek Civil War, was outlawed. In addition, the U.S. backed totalitarian regimes such as the military junta, called the Regime of Colonels, that later ruled Greece.

How often is there an adjustment to the number of representatives in the U.S. House of Representatives that are allotted to each each state? A. Every 10 years. B. Every year. C. Every 20 years. D. Every 100 years.

answer is A. The US Census takes place every 10 years which helps to determine how many people reside in each state. Since representation in the House of Representatives is based on population, this may mean that states are allowed to increase or decrease the amount of people they have based upon the percentage as compared to the whole of the nation

What amendment made poll taxes illegal? A. 24th. B. 15th. C. 17th. D. 22nd.

answer is A. The amendment made it illegal for anyone to try to stop other people from voting by using other means beyond what is legally allowed. This included poll taxes, grandfather clause, and literacy tests

What treaty helped establish the border between Maine and Canada? A. Webster-York Treaty. B. Treaty of Canada. C. Webster-Ashburton Treaty. D. Clay-Calhoun Treaty.

answer is A. The border between the British controlled Canada and American state Maine was disputed for years before the matter was resolved. The famed lawyer Daniel Webster was in charge on the American side for brokering the treaty

Which of the following Ottoman conquests solidified its status as the preeminent power of the eastern Mediterranean and southeastern Europe? A. Constantinople in 1453. B. Chaldiran in 1514. C. Belgrade in 1521. D. Vienna in 1529.

answer is A. The capture of Constantinople in 1453 by the Ottoman Turks began a long period of conquest and expansion, extending its power into Europe and North Africa

What term is used for the total amount of money which is due from the nation? A.) Debt. B.) Deficit. C.) I.O.U. D.) Bill.

answer is A. The debt rises each year as the nation is forced to borrow money not only from citizens but also from other nations around the world. The national debt shows how much has been borrowed without being repaid

What article of the Constitution creates the Judicial branch? A.) Article Three. B.) Article One. C.) Article Four. D.) Article Five.

answer is A. The judges are outlined and it specifically lists what sort of powers they are to have. This is necessary in order to check against the other branches to make sure that they are not allowed to abuse their power and cause harm to Americans.

What dynasty was the last to rule Russia? A. Romanov. B. Hapsburg. C. Kazan. D. Rasputin

answer is A. The last ruling family of Russia, they were driven out and captured by the Russian forces. Plans were abandoned to let them escape to Britain and they were instead executed in a cellar.

Which of the following was suggested by the Virginia Plan? A. That the power of the States should lie within their population. B. That all states should have equal power. C. That all States should be represented both at the State level and in accordance to the size of their population. D. That the States should have limited power

answer is A. The main idea behind the Virginia Plan was that State representation should be determined by the size of the population within a State

All of these are things which can happen to a bill EXCEPT: A. Reiteration. B. Veto. C. Pocket Veto. D. Ratification.

answer is A. The other three are all things which happens to bills after they are passed. Even after a bill is vetoed, it still has a chance of being passed if certain conditions are met

Destroying the rain forests for logging or farming is known as what? A.) Deforestation. B.) Ingraining. C.) Rebuffal. D.) Biodeversity.

answer is A. The process of removing the trees leaves the Earth vulnerable to many natural problems which will occur. The erosion of the Earth happens because the trees are unable to maintain the ground. This also leads to problems known as the greenhouse effect

Which of these mountain ranges would be considered the oldest? A. Appalachian Mountains. B. Rocky Mountains. C. Alps. D. Himalayas.

answer is A. The rounded tops and vegetation growing on these mountains suggest that they are much older than the others. The younger mountains which have recently been pushed together by two tectonic plates are harsher from not having been exposed to erosion for as long as the others.

What term was given to the policy of segregation in South Africa? A. Apartheid. B. Succession. C. Afrikaans. D. Invictus.

answer is A. The segregation was due to the British occupying the nation. It was repealed in 1990 and saw with it the election of Nelson Mandela, the first president of South Africa elected by the population as a whole.

The shifting of a population's economy from agricultural to industrialized is part of which type of economic model? A. Demographic transition B. Primary activity C. Developing transition D. Quinary transition

answer is A. The shift of the world's populations from agricultural to industrialized is a demographic transition. Demography is the study of rates of population change. It encompasses birth and death rates, migration patterns, and population shifts. When a nation shifts to an industrialized economy, the population moves from the country to the cities

What term describes redrawing the judicial lines of a district in order to gain an advantage? A. Gerrymander. B. Salamander. C. Filibuster. D. Redrawstering

answer is A. The term was named for the man who originally tried to create the lines. When running for political office, Gerry redrew the lines of a district in order to win the area. This was found to be unconstitutional and the lines were forced to be changed

The world is divided into climate zones. What are they? A.) Tropical (rainy), dry (desert), humid temperate, humid cold, polar, mountain B.) Tropical (rainy), Tropical (dry), humid, temperate, cold, polar, mountain C.) Equatorial, temperate, dry, polar, mountain D.) Temperate, humid, dry, polar, mountain

answer is A. The world is divided into climate zones with many subdivisions. Factors are humidity, temperature and altitude.

What term is used for the meeting in which either the Democrats or Republicans meet in order to officially announce their candidate for Presidential office? A. National Convention. B. Regional Convention. C. Election Night. D. New Hampshire Primary.

answer is A. These are held so that there will be no mistake as to who is being officially endorsed. It is seen as a place in which all people from the same party are able to meet and discuss topics. Someone cannot officially run as the nominee for their party without getting the endorsement at this event.

What political group was in support of the Constitution being ratified? A. Federalist. B. Anti-Federalist. C. Republicans. D. Democrats.

answer is A. These people wanted the Constitution to be passed so that they would be able to start up the new government. Most of their ideas were for more power to be given to the government than what others wanted

What country, formerly known as Ceylon, was devastated by the tsunami in 2004? A. Sri Lanka. B. Malaysia. C. India. D. Burma.

answer is A. This island nation gained its independence in 1948 from the British Crown and created its own country. There is a strong Buddhist faction on the island.

What fictitious plot to assassinate Charles II eventually led to the arrest and beheading of 35 innocent people? A. Popish Plot. B. Assassination of the Crown Plot. C. Catholic Plot. D.Anti-Crown Plot.

answer is A. This plot was contrived by Titus Oates who gave evidence under testimony in the courts. The plan was to replace Charles II with the Catholic Duke of York. There was never any evidence that this was an actual attempt on the life of the king although people were killed for it

Which of the following statements best describes voter turnout in Presidential elections in the United States since 1996? A.) Despite a general trend toward lower voter turnout since 1960, Presidential elections since 1996 have shown an increase in voter turnout. B.) Despite a general trend toward higher voter turnout since 1960, Presidential elections since 1996 have shown a decrease in voter turnout. C.) Voter turnout in Presidential elections since 1996 continue the general trend of decreasing voter turnout since 1960. D.) Voter turnout in Presidential elections since 1996 continues the general trend of increasing voter turnout since 1960.

answer is A. This question tests your knowledge of current voting trends. According to statistics, the amount of registered voters showing up at the polls has been decreasing since 1960, especially among younger voters. Since 1996, however, this trend has been reversed. Since that time, and particularly in the Presidential elections of 2004 and 2008, the percentage of registered voters getting out to vote has been increasing

China is credited with all of the following inventions, EXCEPT: A. The concept of zero. B. The magnetic compass. C. Woodblock text printing. D. Papermaking.

answer is A. This question tests your knowledge of technological innovation in Asia. The use of the magnetic compass for navigation dates from around 1000 C.E. in China, while the earliest known printed book, from 868 C.E., also comes from China. China is also considered to be the originator of papermaking, with some of the earliest known text-bearing paper going back to 8 B.C.E. The invention of the concept of zero, on the other hand, is credited to India, where it was used in numerical calculations by 1000 C.E., and as a decimal place holder much earlier (around 450 C.E.)

The largest employer in the executive branch of the United States government is the Office of Management and Budget. Which of the following actions best describes the role of that Office? A.) To help the President formulate a budget proposal to submit to Congress for approval. B.) To help Congress verify economic data in the budget proposal submitted by the President. C.) To examine and revise the President's submitted budget proposal in the House, and share revisions with the Senate. D.) To examine and revise the President's submitted budget proposal in the Senate, and share revisions with the House.

answer is A. This question tests your knowledge of the functions of the different branches of government in the budget-making process. In both the Senate and the House—the legislative branch of government—budget committees review the budget proposal submitted by the President, and draft budget resolutions based on the proposed budget. The House version and the Senate version are then reconciled before any specific appropriations bills are drafted. The Congressional Budget Office assists both houses of Congress in this endeavor by providing relevant facts to Congressional committees. The budget process begins, however, with the executive branch, represented by the President. With the assistance of the Office of Management and Budget, the President formulates a budget proposal, and then submits it to both houses of Congress

All of the following are accurate descriptions of Roman law, EXCEPT: A.) In Ancient Rome, the law applied equally to citizens and non-citizens. B.) In Ancient Rome, the law authorized the ownership of slaves. C.) In Ancient Rome, the law granted slaves the right to become free. D.) In Ancient Rome, the law granted slaves the right to own property

answer is A. This question tests your knowledge of the legal system of ancient Rome. In Ancient Rome, slavery was allowed and regulated by law. Under Roman law, slaves could own property given to them my their masters, and they could also be freed by their masters. Freed slaves however, did not have the same rights as men born free in Rome; and non-citizens--including foreigners, did not have the same full privleges and rights under Roman law as full citizens did

What is a slang term for dual federalism? A.Layer cake federalism. B. Onion federalism. C. Split federalism. D. Upside down federalism

answer is A. This refers to the ability that national laws and state laws are different from one another. The national law is on top with the state law being the lower level. They only touch but do not overlap.

The period of time in British history following the return of Charles II as king after Cromwell's rule was known as what? A. Restoration. B Rehabilitation. C Revelation. D Coronation.

answer is A. This time period was marked also with James II as the king. The Anglo-Dutch Wars were also fought during this time period for control of the trading to the New World

What trial dealt with the right to teach evolution in classrooms? A. Scopes Trial. B. Brown vs. Board of Education. C. Darwin Trial. D. Evolution vs. Creation Trial

answer is A. This trial held in Tennessee was over the right for a teacher who wanted to teach evolution and the law which stated that it was illegal to teach this kind of approach to creation. After a length debate in which the judge did not focus on the concept of evolution but rather on the actual law, Scopes was fined $100

What clause was added to the Constitution in which slaves which had escaped to the North had to be returned to the South? A. Fugitive slave cause. B. Escape clause. C. Slavery Clause. D. Return Clause.

answer is A. This was added to make the Southern states ratify the amendment. The Southern states had been deadlocked before this was added because they were worried that once their slaves found out they would be free in the North they would try to escape.

What was the first legislative body in America? A. House of Burgess. B. House of Representative. C. Senate. D. Virginia Town Hall.

answer is A. This was created to give everyone a place to go in order to voice their opinion. It was highly successful and some of the habits learned from there were carried into modern Congress

Striking out a sentence from a bill and then passing it is called what? A. Line veto. B. Pocket passage. C. Presidential veto. D. Rewritten passage.

answer is A. This was declared to be unconstitutional and the bill was not able to be passed. Even today if something is changed in one house after it has passed through another, it must go back to be passed as the exact same bill in both houses.

The act of killing oneself as a samurai to preserve their integrity was known as what? A.) Seppuku. B.) Terrorism. C.) Kamikaze. D.) Shogun.

answer is A. This was done so that the samurai would not have to dishonor his family or himself for crimes he had committed. It was a slow death which was to show the great honor and discipline which a samurai had.

What amendment limits the amount of terms a person can be President? A. 22nd. B. 23rd. C. 21st. D. 11th.

answer is A. This was passed after F.D. Roosevelt served for four terms. While there had not been specifically limits on the amount of terms which could be served, Washington had set the limit by only doing two. This is to make sure that no one person remains in power for such a long amount of time

Measuring changes in the atmosphere and the air is a more in-depth way of looking at what geographical feature? A. Climate. B. Weather. C. Solar Radiation. D. Clouds.

answer is A. To the every day person, climate means expected or habitual weather at a particular place and time of year. To the specialist however, climate is the sum of atmospheric elements (and their variations): solar radiation, temperature, humidity, clouds and precipitation (type, frequency, and amount), atmospheric pressure, and wind (speed and direction)

Well known tropical savanna regions of the world include: A. The Serengeti in Tanzania and the Cerrado in Brazil. B. The Altiplano in Bolivia and Novaya Zemlya in Russia. C. The Gobi in Mongolia and the Atacama in Chile. D. The Congo and Amazon Basins.

answer is A. Tropical (and subtropical) savanna regions are characterized by grasslands with scattered trees and exist at both tropical and subtropical latitudes. They tend to be semi-arid to humid and contain vegetation adapted both to wildfires and grazing animals.

Travelers to Mexico are often warned about "Montezuma's revenge." Who was Montezuma and why is he seeking revenge? A.) The king of the Aztecs killed by Cortes. B.) A god of the Aztecs who wishes to avenge his people. C.) A legendary hero who vanquished the Spanish conquistadores. D.) A Spanish king who hates tourists.

answer is A. When Hernando Cortes arrived in Mexico, he was greeted as a god. He found King Montezuma living in the great city of Tenochtitlan. He captures Montezuma and holds him hostage and kills him. He enslaves the people and eventually destroys the city. Most of the population is killed by Cortes or the diseases which travel with him. Cortes wanted the gold and riches he found with the Aztecs. A great civilization was destroyed

What is the term that means that a region or group in a country has demanded and gained a degree of autonomy or independence from a government? A. Devolution B. Evolution C. Fragmentation D. Perforation

answer is A. When a group or territory successfully breaks away from a government it is called devolution. Evolution is when something evolves or changes. Perforation is a rending or tear. Fragmentation is to have parts. The U.S. is a fragmented nation because Alaska and Hawaii are separated from the main territory

The surrender of Confederate forces to Grant's Union army at Vicksburg was crucial for which of the following reasons? A.) The North had control of the Mississippi River. B.) The North had control of seaports on the coast of Georgia. C.) The North was able to free significant numbers of slaves who joined the Union Army. D.) The North gained control of the coal mines of West Virginia

answer is A. With control of the Mississippi River, the North had cut off a major supply route for Confederate forces west of the Mississippi as well as those north of Vicksburg

The Sino-Japanese War of 1894-1895 resulted in the independence of which of the following countries? A. Korea. B. Manchuria. C. Taiwan. D. Mongolia

answer is A. With the industrial militarization of Japan during the Meiji period, Japan desired to have an open trade relationship with Korea. To do this, Korea could either become a formal dependency of Japan--that is--a colony, or a commercially-dependent trading partner. First, however, the influence of China would need to be curtailed in the region. Japanese troops entered Korea in order to undermine the existing pro-Chinese government, and Japanese forces also attacked Chinese interests in Manchuria and Taiwan. Japan, with technological and strategic advantages, defeated China, forcing China to sign the Treaty of Shimonoseki which recognized Korea as an independent nation.

Which of the following is the degree to which an individual's, family's, or group's social status can change in a given society? A.Social stratification. B. Social mobility. C. Social conformity. D. Social deviance.

answer is B Social mobility, the degree to which an individual can move through the class system, results directly in social stratification.

The capital of Saudi Arabia is: A. Jizan. B. Riyadh. C. Dubai. D. Jeddah

answer is B--Riyadh. In addition to being the seat of the royal government, Riyadh, with a population of more than 5 million, is also Saudi Arabia's largest city. Jizan is a city in southwestern Saudi Arabia, near the border of Yemen, on the Red Sea coast. Jeddah is also on the Red Sea coast, but further to the north, just west of Mecca. Dubai, on the other side of the Arabian Peninsula on the Persian Gulf, is one of the seven city-states that make up the United Arab Emirates

John Locke presented ideas that included the concepts of removing an unjust ruler from power and rulers receiving the right to govern directly from the people. Based on this information, which of the following LEAST represents the ideas of John Locke? A.) Republic. B.) Dictatorship. C.) Constitutional democracy. D.) Representative democracy.

answer is B. A dictatorship is a government in which the people have absolutely no power, and is an example of a form of leadership that would have been opposed by Locke. In both forms of democracy, the voices of the people are heard, and in a republic, government is based on popular representation.

Meeting a neighbor while taking the trash to the curb is an example of which of the following social interactions? A. Accidental. B. Repeated. C. Regular. D. Regulated.

answer is B. A repeated social interaction is not planned, but it is bound to happen from time to time, such as meeting a neighbor when stepping outside

"Mahiz" is a word used by the Taino Indians for the what became known as "maize" by the Spaniards. We call it corn. Why was corn so important when it came to Europe from the New World? A. It was tasty. B. It grew well there and became a staple. C. It replaced potatoes as a staple. D. It was easy to store.

answer is B. A staple is a food you can rely on. Corn grew well and is easy to store.Rye, wheat and rice are also easy to store and were already grown in Europe. Wheat needs good soil and rice needs lots of water. Corn would grow in poorer soil. In England, all cereal plants are called "corn." Potatoes came from the New World too

Which of the following were the MAJOR target of the Crusades? A.) Jews. B.) Muslims. C.) Russians. D.) Mongols.

answer is B. Although the Crusades did direct some attention to the Jews, Russians and Mongols, their original purpose was to recapture Jerusalem and the Holy Land from Muslim control

Trial lawyers must inform the opposition of any witnesses they plan to call. Why is this so? A.) A defendant has the right to a swift trial. B.) A defendant has the right to be confronted with the witnesses against him. C.) Lawyers like to bring in surprise witnesses at the last minute to confound the opposition. D.) Judges don't like surprises.

answer is B. Amendment VI of the U.S. Constitution states that a person has the right "to be confronted with the witnesses against him; to have compulsory witnesses in his favor, and to have the Assistance of Counsel for his defense." The judge and opposing counsel is informed of all witnesses when at all possible

"A state...can be strong but constrained - strong because constrained...Rights to education and other requirements for human development and security aim to advance equal opportunity and personal dignity and to promote a creative and productive society." The ideas expressed in the quote above from sociologist Paul Starr are referring to which of the following? A. Conservativism. B. Liberalism. C. Socialism. D. Nationalism.

answer is B. American liberalism seeks to maximize individual liberty while advocating education, health care and other goods and services believed to be required for human development and self-actualization.

Which type of political leader was Fidel Castro? A. An aristocrat B. An autocrat C. A monarch D. An oligarchy

answer is B. An autocrat is a one-man ruler. A monarch is a hereditary ruler. An oligarchy is a group of people who rule. An aristocrat is someone who inherits power and social standing

Which of the following is addressed by Article Four of the U.S. Constitution? A. Judicial power. B. Power and limits of the States. C. Process of amendments. D. Legislative power.

answer is B. Article Four describes the relationship between the States and the federal government, as well as the relationships among the States

This Spanish explorer started out as a stowaway and ended up establishing the first permanent European colony in the Americas. Who was he? A. Christopher Columbus B. Vasco de Balboa C. Vasco de Gama D. Giovanni Cabuto

answer is B. Balboa stowed away with his dog Leoncico. Before long, he was in command of the ship. He landed in Darien (now called Panama) and established a permanent colony

Which two countries are located within South Africa? A.) Madagascar and Swaziland B.) Swaziland and Lesotho C.) Lesotho and Botswana D.) Casablanca and Lesotho

answer is B. Both these countries are landlocked by South Africa. Casablanca is a city in Morocco. Botswana is a country north of South Africa

Canada is on which of the following continents? A. South America B. North America C. Asia D. Antarctica

answer is B. Canada is on the North American continent, west of Greenland and north of the United States.

When Christopher Columbus struck land, he thought he was in India or Japan. He had landed on an island in the Caribbean inhabited by the Taino people. Soon after this meeting, many of these islanders would be gone from their island. Which is NOT a reason for this depopulation? A.) Columbus and his men kidnapped them for slaves. B.) Columbus put them to work mining gold. C.) They caught diseases from the sailors. D.) They were killed by European weapons.

answer is B. Columbus sought gold but he never found a large amount. Disease, slavery and weapons killed most of the Taino Indians

Andrew Carnegie, John D. Rockefeller, and J. P. Morgan have which of the following in common? A.)They all started out as railroad telegraphers. B.) The foundations of their financial and industrial dynasties were built on meeting the demands of the Union Army during the Civil War. C.) Each of their monopolistic firms in succession was decomposed into smaller companies as a result of Supreme Court cases invoking the Sherman Antitrust Act. D.)Their families were all from Scotland

answer is B. During the Civil War, Rockefeller was involved in supplying kerosene to meet the demands of the Union Army, Morgan's banking firm was involved in meeting the financial demands of the Northern forces, and Carnegie was involved in organizing the army's extensive transportation and communication infrastructure, as well as supplying the growing wartime demand for steel

In the United States House of Representatives, the tenacious task of overt partisan advocacy falls to: A. The Speaker of the House. B. The Majority Leader. C. The Majority Whip. D. The Conference Chair

answer is B. Even though officially the Speaker of the House is the leader of the majority party, he or she tends to take a more non-partisan role, and serves to conciliate the two parties, granting members from both parties the right to speak. Thus it is up to the Majority Leader to aggressively vocalize the majority party's agenda during House of Representatives sessions

In geographical terms, what is a "ped?" A.) A developing country B.) A developed country C.) A foothill of a mountain range D.) A region in the Third World

answer is B. Geographers refer to developed countries as "peds." These are in the First World. The "pings" are the developing countries, also called the Third World

Queen Isabella of Spain helped Columbus begin his famous voyage. What else did Isabella begin about the same time? A. The Spanish Armada B. The Spanish Inquisition C. The Portuguese Wars D. The slave trade

answer is B. In 1492, the Jews were expelled from Spain. Queen Isabella, a Catholic, assembled a religious court called "The Inquisition." It forced non-Catholics to convert or leave Spain. If they refused, they were tortured or executed. Some people objected but were ignored

It is 1684. You are a slave working in the home of Elder Jones in New York City. How has African slavery expanded in New York? I. The slave trade from Africa has expanded bringing more slaves. II. The children of slaves were automatically slaves. III All black people were automatically slaves. IV. New York's legislature recognized slavery as a legitimate practice. A.) I and II only B.) I, II and IV only C.) I, II, III only D.) IV only

answer is B. In 1684, New York declared slavery as a legitimate practice. In 1662, Virginia declared that the children of slaves were automatically slaves. If you had been sold from Virginia, you would be a slave in N.Y.if you had been born there. In 1664, Maryland passed a law that all black people were slaves for life. This was not true in all the colonies. Some slaves could buy or earn their freedom

In response to Japan's occupation of Indochina in 1940, the United States did which of the following? A.) Declared war on Japan. B.) Froze Japanese assets in the U.S. C.) Signed the Tripartite Pact. D.) Embarked upon the Bataan March in the Philippines

answer is B. In addition to freezing Japanese assets in the U.S., the United States also finally decided—after ten years' of Japanese aggression in Asia and the Pacific—to stop selling oil and scrap metal to Japan. Rather than halting Japan's expansion, however, this imposed scarcity of strategic matériel only caused Japan to invade other islands in the Pacific.

The fact that the different states of Mexico have their own constitutions, while the different counties, or municipios (municipalities), of each state do not, implies that: A.) Mexico is a confederation of sovereign anarchist states. B.) Mexico has a federal form of government, while the Mexican states have unitary forms of government. C.) Mexico has a unitary form of government, while the Mexican states have federal forms of government. D.) Mexico is an empire of sovereign anarcho-syndicalist nations.

answer is B. In federal forms of government, such as that of Mexico or the United States, the governments within each individual state do not have the same degree of sovereignty as the state. This sovereignty is represented by the state constitutions. Instead, the state governments are unitary, and power devolves from the central government to the units within the state, such as counties and municipios (municipalities).

Which of the following is famous for the Theory of the Social Contract? A. Karl Marx. B. John Locke. C. Thomas Hobbes. D. Niccolo Machiavelli.

answer is B. John Locke, considered one of the most influential thinkers of the Age of Enlightenment, is responsible for coining the term, Social Contract.

John Winthrop was the leader of this early American group which had an enormous impact on the America of today? What group was it? A. The Pilgrims B. The Puritans C. The Quakers D. The Mormons

answer is B. John Winthrop lead a group of Puritans to Massachusetts Bay in 1630. The charter they used has had a big impact on roots of our modern corporate system, representative form of government, the American legal system and the moral conflict between rights and responsibilities.

The location of the West African cities Mali and Ghana encouraged their growth because: A. They had a wealth of agriculture. B. They were located on the trade routes. C. They were safely nestled in the mountains. D. Mali and Ghana were not influenced by their location.

answer is B. Mali and Ghana became quite wealthy as a result of being located directly on the trade routes.

The Allied invasion of Nazi-Occupied France needed to be carefully planned and precisely timed, not just to coordinate the massive numbers of troops involved, but also because: A.)The Germans had decoded the Allied secret messages. B.) The invasion needed to take place at high tide on a full-moon night. C.) Russia was also planning an invasion at the same time on the Eastern Front. D.) It needed to coincide with the Allied invasion of Japan

answer is B. On June 6, 1944, the Nazi forces had occupied France for four years and had had ample time to fortify the coast with sunken defensive devices intended to thwart invasion by crippling ships and making it difficut to get to shore. In order to get landing craft close enough to the coast to unload troops and tanks, the invasion had to coincide with a high tide so that the boats could float above the obstacles. The highest high tides, or spring tides, take place during both the new moon and the full moon. The Allies needed the latter, however, since several nighttime landings of paratroopers and gliders were also planned, as well as bombing forays inland, and pilots needed the light of the full moon in order to see the way to their targets

Which of the following American authors was particularly influential on the poltical tactics of Mahatma Gandhi and Martin Luther King, Jr.? A. Louisa May Alcott. B. Henry David Thoreau. C. Emily Dickinson. D. Stephen Foster.

answer is B. On the Duty of Civil Disobedience (1849) by Henry David Thoreau influenced the tactics of civil disobedience and passive resistance used by both Gandhi and King

The Communist Revolution in China consolidated power in which of the following ways? A. Depriving women of the right to divorce. B. Destroying the family-centered values of Confucianism. C. Imposing the practice of footbinding on rural communities. D. Encouraging children to marry at a young age

answer is B. One of the Communists' primary strategies in gaining political power in China, was to undermine the 2,000-year-old tradition of Confucianism. In Confucianism, power is vested in the father, and patriarchal lineages in general, and all relationships, including bureaucratic relationships, flow out of these. The Communists set out to destroy this system of relationships and power, so that families and classes would not be in a position to compete for power with the state, but the state would be an individual's only family

Which of the following was an important factor in the collapse of the Romanov Dynasty in early 1917? A. Bolshevik Revolution. B. Food shortages C. Peasant uprising. D. Vladimir Lenin.

answer is B. Russia was suffering from severe food shortages, and these, combined with the heavy losses suffered in the War against Austria-Hungary, led to the Tsar's abdication and Russian withdrawal from the front. The Bolsehvik Revolution only took place in 1918, with Lenin overthrowing Kerensky's provisional Soviet government which had replaced the Imperial government. Between the time of Kerensky's installment as provisional leader and Lenin'e appropriation of power, peasants did revolt, clamoring especially for the cessation of Russian involvement in the War.

The slash-and-burn method of agriculture is best known as part of which of the following? A. The Ice Age. B. The Stone Age. C. The Bronze Age. D. The Paleolithic Age.

answer is B. Slash-and-burn agriculture is literally the cutting down and burning of forests to transform the land into suitable pasture and crop land. This is the first type of agriculture in written history and is most often associated with the Stone Age because at the end of the preceding Ice Age, prehistoric humans lost many of their game and turned to agriculture

In 1927, Gutzon Borglum began work on a giant sculpture to honor which of the following? A. George Washington B. George Washington, Abraham Lincoln, Theodore Roosevelt and Thomas Jefferson C. George Washington, Abraham Lincoln, Franklin Roosevelt and Thomas Jefferson D. "Buffalo" Bill Cody

answer is B. South Dakota state historian Doane Robinson loved the Wild West. He wanted to build a monument to its heroes, like Buffalo Bill. However, when he contracted (John) Gutzon de la Mothe Borglum to do the sculpting, Borglum convinced Robinson that sculptures of a more national character would be more broadly popular and enduring. Borglum's Mount Rushmore project occupied him from 1927 until his death in 1971.

The Spanish might be blamed for the Irish potato famine of the 1800s. How could this be? A.) Many Spanish immigrated to Ireland. B.) Spanish explorers introduced the potato to Europe. C.) The Irish became explorers and brought home the potato. D.) Columbus stopped in Ireland on his way to the New World and brought the virus which caused the potato blight.

answer is B. Spanish explorers helped start an agricultural revolution. They transported plants and animals to and from Europe. They brought potatoes to Europe in the later 16th century. The Irish, which were under the rule of Britain in the 1800s, became so dependent on the potato that many died when the blight killed the crops

You are skiing in the Alps in Europe. You are in a politically neutral country located north of Italy, south of Germany and west of Austria. Where are you? A. France B. Switzerland C. Liechtenstein D. Luxembourg

answer is B. Switzerland is a politically neutral country in the Alps. Part of the Alps is also in Italy. The capital is Bern

Which document served as the first U.S. Constitution? A. The Bill of Rights B.The Articles of Confederation C.The Declaration of Independence D.The "Olive Branch" petition

answer is B. The Articles of Confederation was the nation's first attempt to unite. It provided the nation's framework from 1781 to 1789. It was replaced by the U.S. Constitution

The main reason U.S. forces needed to take over the Japanese island of Okinawa was: A.) To take over the oilfields which were Okinawa's principal export. B.) To secure a staging ground for the planned invasion of Mainland Japan. C.) To build a large enough airfield from which to launch planes carrying atomic bombs. D.) To recruit Okinawans into the Allied forces.

answer is B. The Battle of Okinawa occurred over three months during the spring of 1945. The only other battle of World I War II in which American forces lost more men was at the Battle of the Bulge in northern Europe. The Japanese also suffered tremendous losses, and it is estimated that almost 25% of the civilian population of Okinawa died during the three-month battle. Thus there were very compelling strategic reasons to accept such immense destruction of life. The guiding vision for the entire U.S. strategy of "island hopping"—taking Japanese controlled territory island by island rather than simply bombing from aircraft carriers and U.S. territory—was the invasion of Japan which had been in the planning stages since 1943. The U.S. needed to get closer and closer to Japan, and take bigger and bigger islands, in order to have ample staging grounds from which to launch a massive invasion force as had been done at Normandy on June 6, 1944. Since Okinawa is only 340 miles from Japan

Benjamin Bache wrote" if ever a nation was debauched by a man, the American nation has been debauched by Washington." His statement, printed in the nation's newspapers was possible because of what? A. The printing press B. Freedom of the press C. Washington's wild lifestyle D. Thomas Jefferson's backing

answer is B. The Bill of Rights guarantees "freedom of speech, or of the press;" Newspapers have a right to publish opinions as long as they are not libelous

The British ruled the Asian sub-continent from 1858 to 1947. After independence, they partitioned the sub-continent into two nations. What are they? A. India and Afghanistan B. India and Pakistan C. India and Kashmir D. India and Nepal

answer is B. The British divided the continent into India and Pakistan without taking into consideration the many tribal loyalties which existed. The area of the Kashmir is still being disputed

When the war between the British and the Americans started in 1775, the British had a standing army and a fine navy. The Americans had neither. Which did they have that gave them some advantage in the war? A.) An agreement with the French for ships and troops B.) No central government C.) A system of currency D.) Well trained militiamen

answer is B. The British had a well-trained standing army and employed foreign mercenaries. They had the best navy in the world. The colonists had no regular army, no navy and few resources to assemble them. But they did know the land and had a cause the believed in. Because they had no central point of power, the British had to conquer each city one at a time. So, for example, if they took Philadelphia, New York might still be in colonial control. Their system of currency was basically worthless

hat land form runs from north to south for hundreds of miles in the eastern region of Africa? A. The Sahara desert B. the Great Rift Valley C. The Namib Desert D. Lake Victoria

answer is B. The Great Rift Valley runs for hundreds of miles from north to south in the eastern region of Africa. The Sahara Desert stretches across northern Africa. The Namib desert is in western Africa. Lake Victoria is the world's third largest lake

The number of justices on the Supreme Court is determined by: A. Executive Order. B. judicial act. C. judicial decree D. Article III of the Constitution.

answer is B. The Judiciary Act of 1869, passed by both houses of Congress on April 10 of that year, states that "the Supreme Court of the United States shall hereafter consist of the Chief Justice of the United States and eight associate justices, any six of whom shall constitute a quorum."

The Orinoco River flows through which of the following countries? A. Brazil and Bolivia. B. Venezuela and Colombia. C. Cameroon and Gabon. D. Thailand and Cambodia.

answer is B. The Orinoco River is 1,330 miles long and crosses the entire country of Venezuela east to west. About a quarter of the river's length is in Colombia.

Which of the following Acts is held widely responsible for the onset of the Great Depression? A. Davis-Bacon Act. B. Smoot-Hawley Tariff Act. C. Agriculture Marketing Act. D. Reapportionment Act of 1929.

answer is B. The Smoot-Hawley Tariff Act raised tariffs on over 20,000 imported goods to record levels. Many countries retaliated with their own increased tariffs on U.S. goods, and American exports and imports plunged by more than fifty percent. That is the reason why the Smoot-Hawley Tariff Act is considered to be at least partially responsible for the Great Depression.

The world's first artificial satellite was launched by: A. The United States in 1955. B. The Soviet Union in 1957. C. China in 1958. D. France in 1960.

answer is B. The Soviet Union launched Sputnik 1 on October 4, 1957. The United States started Project Vanguard in 1955, and succeeded in launching Explorer 1 on January 31, 1958. France launched a satellite in 1965, and China lauched one in 1970

Charles Townshend, known as "Champagne Charley" the British Prime Minister in 1767, had an act of Parliament named after him. He had vowed to "squeeze" some money out of the American colonies to pay British debt. What did his act tax? A. Wine B. Glass C. Silk D. Playing cards

answer is B. The Stamp Act taxed almost every legal document as playing cards. In some cases it doubled their cost. The Townshend Act taxed tea, glass, and paper, among other things. The Proclamation of 1763 banned westward expansion. The Revenue Act raised taxes on linen, wine, sugar, and silk. The British had overextended themselves fighting many wars and needed money. They taxed the colonies to help defray this because the cost of protecting the colonies was part of this debt

The Supreme Court decision of Helvering v. Northwest Steel Rolling Mills, Inc. (1940) ruled that the Revenue Act of 1936 did not violate the Constitution because it did not infringe "the reserved powers of the States over corporations—to prescribe their powers and condition the exercise thereof." The phrase "reserved powers of the States" refers to which of the following constitutional amendments? A.) Fourth. B.) Tenth. C.) 11th. D.) 15th.

answer is B. The Tenth Amendment reads in full: "The powers not delegated to the United States by the Constitution, nor prohibited by it to the States, are reserved to the States respectively, or to the people."

The Voting Rights Act of 1965 had which of the following effects? A. Granted all United States citizens the right to vote. B. Outlawed poll taxes and literacy tests. C. Outlawed ID requirements. D. Ruled that voting places must be wheelchair accessible

answer is B. The Voting Rights Act of 1965 did not grant the right to vote to all United States citizens (for example, in many states, people in prisons and ex-felons do not have the right to vote), nor did it outlaw the requirement to show some form of identification in order to prove residency that some states have. The Act also did not touch on accessibility requirements for people in wheelchairs or with other mobility impairments. Rather, the Votingl Rights Act of 1965 specifically outlawed the poll tax and literacy test that several southern states had used to disenfranchise Black voters.

Which of the following was a Democratic bill that reversed high tariffs imposed by the Whig Party? A. Black Tariff. B. Walker Tariff. C. Morrill Tariff. D. Tariff of Abominations.

answer is B. The Walker Tariff was one of the lowest tariffs in American history, and was primarily supported by Southern Democrats who had little industry in their districts.

In the United States, only citizens have the right to: A.Own a house. B. Vote. C. Work. D. Protection by the police.

answer is B. The concept of citizenship functions to regulate identity with and participation in a society. In the United States, the rights of citizenship allow one to participate in the workings of government by voting, but non-citizens, who are deprived of the right to vote, are allowed to participate by working or by purchasing a house. In addition, everyone in the country, whether a citizen or not, has the right to the security provided by the police, as the 14th Amendment to the Constitution provides: "No state shall... deprive any person of life, liberty, or property, without due process of law; nor deny to any person within its jurisdiction the equal protection of the laws."

During the Heian Period, diaries such as the Pillow Book and novels such as the Tale of Genji were written by: A. Japanese noblemen. B. Japanese noblewomen. C. Chinese noblemen. D. Chinese noblewomen.

answer is B. The earliest classics of Japanese literature are attributed to the Heian Period (794-1185 C.E.), and they were written by Japanese noblewomen. The men of the Japanese court did write, but as the prestige culture was strongly influenced by China, they wrote poems in Chinese. The Pillow Book was written by Sei Shonagon, a lady-in-waiting of the Japanese Empress, and the Tale of Genji by Murasaki Shikibu, an imperial maid of honor.

Mount St. Helens in the United States, Mount Merapi in Indonesia, Mount Fuji in Japan, Mount Ruapehu in New Zealand, and Cerro Arenal in Costa Rica are: A.) The five highest mountains in the world. B.) Volcanoes located along the Pacific Ring of Fire. C.) Covered in snow throughout the year. D.) Important pilgrimage sites along the Pacific Protestant Trail.

answer is B. The majority of the world's volcanoes exist along the Pacific Ring of Fire, due to the subduction and uplift occurring at the boundaries of tectonic plates. This extensive region of seismic activity resulting in volcanism and earthquakes can be traced on a map along the western coasts of South and North America, across the northern Pacific by following the islands of the Aleutians, then south along the eastern coasts of Japan and down to the Philippines and Indonesia, curving to the northwest of New Guinea, and plunging south again to New Zealand.

The Second Continental Congress met in Philadelphia in 1776. What is a "Congress?" A.) A group of statesmen gathered to discuss the law B.) A group of people selected as representatives of a colony or state C.) A group who decided where the capital of the nation would be D.) A group organized by a leader to advise him/her

answer is B. The members of the Second Continental Congress were selected by each colony's legislature to represent them. Today we elect members to Congress.

"A prince ought to have no other aim or thought, nor select anything else for his study, than war and its rules and discipline; for this is the sole art that belongs to him who rules, and it is of such force that it not only upholds those who are born princes, but it often enables men to rise from a private station to that rank." The preceding passage most closely reflects the ideas of which of the following political theorists? A. Thomas Hobbes. B. Niccolò Machiavelli. C. Friedrich Nietzsche. D. Sun Tzu.

answer is B. The passage is quoted from Chapter 14 of Niccolò Machiavelli's Prince.

One factor that enabled the transition from feudalism to market economies in Europe was: A. The code of chivalry. B. The three-field system of farming. C. Primogeniture. D. The introduction of Arabic writings on algebra and geometry

answer is B. The three-field system of farming made food production more efficient on the feudal manors. By keeping one field fallow, while two others were seasonally split between a fall and a spring harvest, the manors built up agricultural surpluses. These surpluses then allowed certain serfs to take time away from farming and develop specialized craft skills, the products of which could be exchanged with other manors, and other parts of Europe. This trade in specialized craft products laid the foundation for a market economy as well as for the later guild system of craftsmen.

In 1947, India became an independent dominion of the British Commonwealth, while in 1950 it became a democratic republic with its own Constitution. Which of the following foreign policy approaches best describes that from which India gained independence? A. Isolationism. B. Imperialism. C. Interventionism. D. Internationalism.

answer is B. This question tests your ability to relate real world events to different foreign policy approaches. Isolationism espouses a policy of avoiding wars with other countries except in cases of self-defense, and erecting economic barriers controlling trade with, and cultural influences from, other countries. Internationalism views nations as potentially equal players in the realm of foreign policy, and urges them to cooperate economically and politically for the benefit to all and cost to none. Interventionism is the policy by which one country manipulates another through either economic means such as trade sanctions, embargoes, export limitations, military means such as overt or clandestine operations, assassinations, terrorism, etc. Imperialism is a form of large-scale intervention by which one country controls another politically, militarily, culturally, and economically by formally claiming it as a dependent protectorate or colony. Since India had been such a dependent colony of Britain, the foreign policy from which it gained independence most closely approaches imperialism

Although the religion known as Buddhism started in India, it is not very common there. Instead, Buddhism is more popular in other parts of Asia, including China, Thailand, and Vietnam. The process that best accounts for this widespread distribution is known as: A. Cultural innovation. B. Cultural diffusion. C. Social stratification. D. Social mobility.

answer is B. This question tests your knowledge of a key factor of cultural change. Buddhism's start in India is an example of cultural innovation. However, Buddhist monks spread their religion to other countries and cultures of the world. Since both social mobility and social stratification involve change and distribution only within a particular culture, the process that best explains the spread of change to other cultures is called cultural diffusio

When were The Great Pyramids of ancient Egypt built? A. In the Neolithic Period. B. In the Old Kingdom. C. In the New Kingdom. D. In the Iron Age

answer is B. This question tests your knowledge of ancient Egypt. The Great Pyramids were built as royal tombs in the Giza Necropolis outside the capital city of Memphis (the outskirts of present-day Cairo) during the time of the Old Kingdom (2686-2134 B.C.E.). The Valleys of the Kings and Queens are sites of royal necropoli containing tombs built further south along the Nile outside the city of Thebes--capital during the time of the New Kingdom (1570-1070 B.C.E.)

The tropical regions of the world are most prevalent near: A. The Poles. B. The equator. C. The prime meridian. D. The International Date Line.

answer is B. This question tests your knowledge of how the Earth's latitude and longitude relate to its climatic features. Latitude strongly correlates with climate, while longitude does not. The tropical regions of the world are typically the warmest regions because they receive constant sunlight year-round. The region of the Earth that receives the most constant sunlight lies along the equator. The Poles by contrast, receive direct sunlight only half of the year, and therefore are very cold. The prime meridian and International Date Line, consist of lines of longitude, and do not correlate with climate because by definition they are markers of time and not regions or masses of land in particular

When did the empire of ancient Rome have its greatest extent? A.) When it was ruled by Tiberius in 17 C.E. B.) When it was ruled by Trajan in 117 C.E. C.) When it was ruled by Septimius Severus in 197 C.E. D.) When it was ruled by Diocletian in 297 C.E.

answer is B. This question tests your knowledge of the history of the Roman Empire. The Roman Empire began in 27 B.C.E., with the reign of Octavian, also known as Caesar Augustus. Around 476 C.E., the Empire split into western and eastern halves, with the latter evolving into the Byzantine Empire. This Eastern continuation of the Christian Roman Empire did not collapse until 1453 C.E., when Constantinople was conquered by the Ottomons. Long before this date, the Roman Empire's period of maximal external expansion and minimal internal strife, known as the Pax Romana, reached its peak during the reign of Trajan, from 98 to 117 C.E. The Pax Romana continued through the reign of Marcus Aurelius, whose death in 180 C.E. marked the beginning of the Empire's contraction and fragmentation

In the 1770s, the colonies grew restless. Anger at England caused some incidents in Boston. What happened on when some malcontents began to throw rocks and snowballs at a British sentry outside the customs house? A.) The Boston Tea Party B.) The Boston Massacre C.) The burning in effigy of Gov. Hutchinson D.) The Intolerable Acts

answer is B. When the crowd began throwing rocks and snowballs at the sentry, another 20 soldiers appeared. Someone fired into the crowd and 11 members of the mob were wounded or dead, including Crispus Attucks, a black man, the first man to die in the Revolution. The soldiers were put on trial and defended by John Adams who took the case even though no one thought it was prudent. Radicals in Boston called this "The Boston Massacre." The soldiers were mostly acquitted with a few being branded as a punishment

Before the invention of movable type, most books were copied by hand. What were these books called? A. Magazines B. Newspapers C. Manuscripts D. Handscripts

answer is C. A scribe worked copying a manuscript, Manuscript comes from the Latin for "hand" and "writing."

Which of the following statements most accurately describes the outcome of the 1796 presidential election? A.) John Adams was the Federalist Party Presidential candidate, and Thomas Jefferson was the Federalist Party Vice-Presidential candidate; Adams was elected President, and thus Jefferson became Vice-President. B.) Thomas Jefferson was the Federalist Party Presidential candidate while John Adams was the Presidential candidate for the Democratic-Republican party; Jefferson was elected President, and Adams was elected Vice-President. C.) John Adams was the Federalist Party Presidential candidate while Thomas Jefferson was the Presidential candidate for the Democratic-Republican party; Adams was elected President, and Jefferson was elected Vice-President. D.) Thomas Jefferson was the Federalist Party Presidential candidate and John Adams was the Federalist Party Vice-Presidential candidate; Jefferson was elected President, and thus Adams became Vice President.

answer is C. Adams and Jefferson both ran for President, each for a different party. Adams was the candidate for the Federalist Party, while Jefferson was the candidate for the Democratic-Republican Party. As Adams received the greatest number of electoral votes, he became President. Adams' opponent, Jefferson, received the second greatest number of votes, and thus became Vice-President

Which of the following is the largest body of water indenting North America? A.The Gulf of St. Lawrence. B. Hudson Bay. C. The Gulf of Mexico. D. The Gulf of California.

answer is C. Although Hudson Bay has the largest shoreline in North America, the Gulf of Mexico is the largest body of water

Which of the following statements is FALSE concerning the Columbian Exchange? A.) The Columbian Exchange deeply affected worldwide society. B.)The Columbian Exchange was indirectly related to the Irish Potato Famine. C.) The Columbian Exchange caused a long-term decrease in the world population. D.) The Columbian Exchange took place after Christopher Columbus arrived to the Americas

answer is C. Although the Columbian Exchange brought deadly diseases that initially reduced population, over the long term, the exchange of ideas, crops and livestock actually increased the world population. "The Columbian Exchange caused a long-term decrease in the world population." is the only false statement. The Columbian Exchange, the widespread exchange of plants, animals, cultures, and populations after Columbus's voyage to the Americas, caused dramatic changes in world culture. Prior to the Columbian exchange, potatoes were were only grown in South America. The post-Columbian exchange of plant crops introduced potatoes to Europe, and by the time of the Irish Potato Famine in 1800, Ireland was almost completely dependent on the potato as a food source

By the end of the 17th century, slavery was expanding rapidly in the colonies. What is one reason for this? A. The colonists needed skilled workers. B. The indentured servant practice was waning. C. Slavery was more economic than indentured servants. D. The English had banned the slave trade

answer is C. America needed labor. European indentured servants, workers who were only in service for a given period of time, had to be housed, fed and paid according to European standards. Slaves did not have any economic rights and were cheaper to use

Who was the first president to have impeachment charges brought against him? A. Clinton B. Cleveland C. Johnson D. Nixon

answer is C. Andrew Johnson violated the Tenure of Office Act when he attempted to remove Secretary of War, Edwin Stanton, from office without the approval of the Senate. He was acquitted because the senate lacked the necessary 2/3 majority for a "guilty" verdict, and he remained in office.

The U.S.Constitution gives final appellate jurisdiction to which branch of government? A. The Federal Courts B. The Appellate Courts C. The Supreme Court D. The Congress

answer is C. Article III of the Constitution gives the right to final appeal to the Supreme Court. Other courts have other jurisdictions inferior to the Supreme Court

The term of office for a United States Senator is: A. Two years. B. Four years. C. Six years. D. Eight years.

answer is C. As Article I, Section 3 of the Constitution states: "The Senate of the United States shall be composed of two Senators from each State, chosen by the Legislature thereof, for six years; and each Senator shall have one Vote."

Which of these was NOT invented by Benjamin Franklin? A. Franklin stove. B. Bifocals. C. Gas lamps. D.Lightning rod.

answer is C. Being an inventor, Franklin soon caught the attention of the nation as well as the world. He would go on to become one of the Founding Fathers of America

Nicholas Biddle is responsible for helping create what centralized government service? A. Postal service. B. Road service. C. Bank service. D. Train service.

answer is C. Biddle was appointed by the president to the office of president of the Second United States bank and noticed the need for a central bank to exist. Thanks to him, all banks function under the same regulations. Jackson tried to do away with this later in his term but was outvoted.

Which units of measurement may be used as part of the scale on a map? I. Miles II. Inches III. Meters IV. Leagues A. I, II and III only B. I and III only C. I, II, III and IV D. None of the choices is correct

answer is C. Cartographers, or map makers, use scale, the ratio of the distance on the map and the corresponding distance on the surface of the earth, when making maps. Maps are drawn to a reduced scale or they would be as large as the area being represented. A map's scale is expressed as a ratio, such as 1 inch:20,000 miles. Any unit of measurement may be used. A bar scale is often placed on the map to help the reader.

Which of the following did Erik Erikson classify as the major developmental task of adolescence? A. Intimacy. B. Integrity. C. Identity. D. Responsibility.

answer is C. Erikson's theory of psychosocial development suggests that during adolescence, people are struggling with the crisis of Identity vs. Role Confusion. Intimacy, integrity and responsibility are all involved in other stages of Erikson's theory

Which of the following was the earliest successful British colony in North America? A. Plymouth Colony. B. Roanoke Colony. C. Jamestown Colony. D. Delaware Colony.

answer is C. Established in 1587, the Roanoke Colony in Virginia is the earliest British colony in North America, but it disappeared under mysterious circumstances. The order of establishment of the succeeding—and succesful—British colonies was Jamestown in 1607 and Plymouth in 1620. Delaware Colony, settled by the Dutch in 1631, only came under British control after the Duke of York's victorious siege of New Amsterdam in 1664

What country originally settled Manhattan? A. France. B. Spain. C. Holland. D. England

answer is C. For a few beads, the Dutch were able to convince the Indians to sell them the island of Manhattan. They established New Amsterdam which was later renamed New York

Fighting between Americas and Native Americans heated up in Florida during what? A. Native American Uprising '42. B. American-Indian War. C. Seminole War. D. Braves War.

answer is C. Fought in Florida, the war was to convince the Native Americans to leave the land so that settlers could begin to cultivate it. After three times fighting the Native Americans, Congress passed legislature forcing all of the Seminoles and other Native Americans from that area to leave. This long journey was later known as the Trail of Tears.

The Spanish Inquisition began in 1492. How long did it last? A. 100 years B. 30 years C. Over 300 years. D. 200 years

answer is C. In 1492, the Jews were expelled from Spain. Queen Isabella, a Catholic, assembled a religious court called "The Inquisition." It forced non-Catholics to convert or leave Spain. If they refused, they were tortured or executed. The Inquisition spread to Peru and Mexico and did not end until 1834, over 300 years later

Which of the following pair of diseases did early European sailors take to, and bring back from the Americas? A. Bubonic Plague—Dengue Fever. B. Ebola—Trypanosomiasis. C. Smallpox—Syphilis. D. Yellow Fever—Malaria.

answer is C. In the New World, the historical epicenters of smallpox infection tend to cluster around the sites of first landfall. This is one reason why the indigenous populations of countries such as Cuba and Costa Rica—sites of some of the earliest Spanish exploration—today are far less in number than in countries such as Guatemala and Ecuador. In addition, scholars believe that sailors aboard the ships in Columbus' first voyage contracted syphillis from the native peoples of the Antilles, then initiated the spread of the Disease throughout the Old World when they served in various European wars

What President also wrote the Declaration of Independence? A. Abraham Lincoln. B. Benjamin Franklin. C. Thomas Jefferson. D. George Washington

answer is C. Jefferson was both a statesman and an accomplished writer when he set out to write the Declaration of Independence. Many of the ideas set forth in it are from prominent British thinkers

Learning table manners from parents is an example of which of the following? A. Rote learning. B. Formal learning. C. Informal learning. D. M-learning.

answer is C. Learning that occurs as a result of being involved in day-to-day situations, such as table manners while eating a meal, is known as informal learning

You are in Greenwich England. What line of longitude are you standing on? A. 60 degrees longitude B. 90 degrees longitude C. 0 degrees longitude D. 45 degrees longitude

answer is C. Lines of latitude are also called parallels and run east-west. They are parallel to the equator. Lines of longitude are called meridians and run north-south. The equator is 0 degrees latitude. The prime meridian is 0 degrees longitude. It runs through the Royal Observatory at Greenwich, England. The distance from the prime meridian sets the time zones

During the Renaissance, explorers all wanted to get to the Orient. Why? A. They wanted to be the first to go there. B. It had not been colonized yet. C. It had spices, gold and jewels. D. They could make money from mapping the area

answer is C. Marco Polo had chronicled his trip to the Orient in the 13th century and told of the riches there. He had brought back samples. Everyone wanted to get there to become rich and famous

Prince Henry of Lisbon wanted his sailors to travel to the Orient. Why did he need sailors? A.) He wanted to get there first. B.) He wanted to cash in on the ship building industry. C.) The land routes were dangerous and difficult. D.) The land routes were needed to be mapped.

answer is C. Marco Polo had chronicled his trip to the Orient in the 13th century and told of the riches there. He had brought back samples. Everyone wanted to get there to become rich and famous. Prince Henry wanted his sailors to sail around Africa to avoid the dangers of the land journey. Rival Muslim empires were warring for control there

Gerardus Mercator was a Flemish cartographer who developed a map or projection of the world. What else did he develop? A.) A globe B.) A system of directions C.) A bound collection of maps called an atlas D.) A topographical map of North America

answer is C. Mercatur's map could be used by sailors to plot a "true" direction. However, his map distorted areas, especially toward the poles. It preserved shape and direction. Atlas was a Greek god who held the world on his shoulders

Which explorer discovered Florida in 1513? A. Francisco Pizarro B. Christopher Columbus C. Ponce de Leon D. Vasco de Balboa

answer is C. Ponce de Leon sailed with Columbus on one of his voyages. He found gold in Puerto Rico and became its governor. He set out to find the fountain of youth and discovered a new land which he named La Florida. Florida means "flowery" in Spanish

In 1608, France was experiencing a period of emigration to the New World. Samuel de Champlain wanted to encourage immigrants to Canada. He founded Quebec but the French Huguenots were not tempted to settle there. Why not? A.) They had a good life in France. B.) They wanted to establish their own colony. C.) They were being persecuted for their religion. D.) They were afraid of the Spanish.

answer is C. Quebec is the second oldest continuously occupied city on the North American continent. Champlain knew that settlements would encourage immigration. Canada was controlled by the Catholics. The Huguenots, being Protestants, would not be welcome there

Many in 16th century Spain were ambivalent about slavery. Two great writers, Juan Sepulveda and Bartolome Las Casas held different views. What were the issues? A.) Slavery was needed if you wanted to get rich. Slavery was evil because the Bible forbade it. B.) Slavery was justified because the conquered were inferior. Slavery was never justified because the Bible forbade it. C.) Slavery was justified because the conquered were inferior. Slavery was due to a greed for riches. D.) Slavery was justified by Aristotle. Slavery was condemned by the church.

answer is C. Sepulvada said that the Indians were "inferior to Spaniards.... like apes to men." Las Casas wrote:" The reason why the Christians have killed so many and destroyed such an infinite number of souls is that they have been moved by their wish for gold and their desire to enrich themselves." Aristotle justified slavery. The Bible both condoned and rejected it.

In parliamentary systems of government, the prime minister may be removed from office by which of the following means? A.Referendum. B. Impeachment. C. Vote of no confidence. D. Amendment to the Constitution.

answer is C. Since the Prime Minister is typically appointed by members of the legislature, a vote of no confidence in the legislature will serve for the Prime Minister's resignation, or, sometimes, the dissolution of Parliament. In 1974, Canadian Prime Minister Pierre Trudeau intentionally got himself removed from office by failing to pass the budget. By doing so, he sped up the date of elections so that his party could win a majority of seats, and thereby he was re-appointed Prime Minister with a government more favorable to his policies

What general in the Civil War become President? A. Lincoln. B. Cleveland. C. Grant. D. Lee.

answer is C. Thanks in part to his leadership skills during the war, Grant was elected by a landslide vote in the North. His main opponents in the South were against him trying to help to rebuild the Southern states in his image.

A local cyclists' rights group, not affiliated with any political party or candidate, makes posters during a Presidential campaign telling voters not to vote for candidate X, because he is against building new bike paths. The group pays $300 to have their posters reproduced, and then they volunteer to hang them throughout their town. Which of the following campaign regulations apply to them? A.) Since they are not affiliated with any candidate or party, they can print as many posters as they want and pay as much as they are able to; and will not have to report to the FEC. B.) As long as there are at least two members in the cyclists' rights group involved in paying for and hanging the posters, they will not have to report to the FEC, since their individual expenses are less than $250. C.) They will need to report their $300 expense to the Federal Election Commission, since they have paid more than $250. D.) Since they are volunteers, and the money is coming out of their own pocket, they do not need to report the expense to the FEC, and they can write off their expenses on their tax return.

answer is C. The $300 that the cyclists' rights group has paid for printing their posters is considered to be an independent expenditure by the Federal Election Commission. Even though they are not affiliated with any candidate or with any political party, if they are making a statement in favor of one candidate, or against another as a group or as an individual, they need to file their expenses with the FEC, using the special independent expenditures form since it is more than $250. And incidentally--political contributions are not tax-deductible

Which of the following mountain ranges is considered the original homeland of the Turkic people? A. Caucasus. B. Ural. C. Altai. D. Carpathian.

answer is C. The Altai Mountains of Central Asia are located along the borders of northern Mongolia, southeastern Russia, eastern Kazakhstan, and western China

Jan and Joan are running for president of the fourth grade. Jan says the Joan will make recess shorter. Joan says that Jan will make them have more homework. What Constitutional right are the candidates invoking? A. Freedom of assembly B. Freedom of the press C. Freedom of speech D. Freedom of debate

answer is C. The Bill of Rights guarantees "freedom of speech, or of the press;" Newspapers have a right to publish opinions as long as they are not libelous. Candidates can criticize their opponents as long as they do not slander them.

For many years the American colonists had been satisfied with their status with the British. Then something happened which began an period of discontent. What was it? A.) The British began trading more with the Canadian French than the colonies. B.) The British started interferring in local politics. C.) The British wanted the Americans to pay the costs of quartering British soldiers there. D.) The British navy went off to fight a war with France leaving the colonies defenseless.

answer is C. The British had overextended themselves fighting many wars and needed money. The national debt was equivalent to $120 million dollars. They taxed the colonies to help defray this because the cost of protecting the colonies was part of this debt

What space shuttle blew up in 1986 killing all on board it? A. Discovery. B. Apollo. C. Challenger. D. Columbia

answer is C. The Challenger blew up shortly after lift off because of technical errors. All on board, including the first female astronaut, were killed

Which of the following was NOT a direct result of the Columbian Exchange? A. The distribution of goods throughout the world. B. A rise in disease and mortality. C. An increase in isolationism. D. An increase in the slave trade

answer is C. The Columbian Exchange increased trade and communication between countries rather than encouraging isolationism.

Which of the following form the borders of the Dead Sea? A. Lebanon, Israel, Syria. B. Jordan, Israel, Saudi Arabia. C. West Bank, Israel, Jordan. D. Israel, Saudi Arabia, Egypt.

answer is C. The Dead Sea is fed by the Jordan River and its west bank consists of the Palestinian territory of the West Bank and the state of Israel, while its east bank is the state of Jordan.

The holy city of Varansi, or Benares, lies along the banks of which of the following rivers? A. Indus. B. Brahmaputra. C. Ganges. D. Yamuna.

answer is C. The Ganges River in India flows from the Himalayas in the north of India and empties into the Bay of Bengal.

Which of the following was suggested by the Great Compromise of 1787? A.That the power of the States should lie within their population. B. That all states should have equal power. C. That all States should be represented both at the State level and in accordance to the size of their population. D. That certain States should have limited power.

answer is C. The Great Compromise of 1787 is largely responsible for determining that in the federal U.S. government, the Senate would represent the States and the House of Representatives would represent the population

The Louisiana Purchase did all of the following, EXCEPT: A. Secured control of North America's longest river. B. Doubled the size of the United States. C. Purchased Florida. D. Added to the Spanish, French, and Native American populations in the United States.

answer is C. The Louisiana Purchase secured control of the Mississippi River; added to the Spanish, French, and Native American populations of the country, and essentially doubled the size of the United States at the time. It did not purchase Florida

"The enumeration in the Constitution, of certain rights, shall not be construed to deny or disparage others retained by the people." The quote above is from which of the following Amendments to the U.S. Constitution? A. Seventh Amendment. B. Eighth Amendment. C. Ninth Amendment. D. Tenth Amendment

answer is C. The Ninth Amendments addresses rights of the people that are not specifically enumerated in the Constitution

The Constitution of the United States provides which of the following instructions regarding the selection of the President? A.) The President shall be elected by obtaining the majority of the popular vote. B.) The President shall be elected by the members of the House of Representatives and the Senate. C.) The President shall be elected by an institution whose members are appointed by the states. D.) The President shall be appointed by a joint committee of the House of Representatives and the Senate

answer is C. The President of the United States is elected by the appointed members of an institution called the Electoral College established by Article II, Section 1 of the Constitution: "Each State shall appoint, in such Manner as the Legislature thereof may direct, a number of Electors, equal to the whole number of Senators and Representatives to which the State may be entitled in the Congress: but no Senator or Representative, or Person holding an Office of Trust or Profit under the United States, shall be appointed an Elector."

The Prime Meridian passes through which of the following countries? A. United States. B. Australia. C. England. D. Colombia.

answer is C. The Prime Meridian passes through Greenwich in East London, England

"In suits at common law, where the value in controversy shall exceed twenty dollars, the right of trial by jury shall be preserved, and no fact tried by a jury shall be otherwise reexamined in any Court of the United States, than according to the rules of the common law." The quote above is from which of the following Amendments to the U.S. Constitution? A.Fifth Amendment. B. Sixth Amendment. C. Seventh Amendment. D. Eighth Amendment.

answer is C. The Seventh Amendment codifies the right to a trial by jury in certain civil cases

"The powers not delegated to the United States by the Constitution, nor prohibited by it to the States, are reserved to the States respectively, or to the people." The quote above is from which of the following Amendments to the U.S. Constitution? A.) Eighth Amendment. B.) Ninth Amendment. C.) Tenth Amendment. D.) Eleventh Amendment.

answer is C. The Tenth Amendment specifically addresses the balance of power among the States, the federal government, and the people

"No Soldier shall, in time of peace be quartered in any house, without the consent of the Owner, nor in time of war, but in a manner to be prescribed by law." The quote above is from which of the following Amendments to the U.S. Constitution? A. First Amendment. B. Second Amendment. C. Third Amendment. D. Fourth Amendment.

answer is C. The Third Amendment was written to avoid the recurrence of quartering in citizens' homes as had been done before the American Revolution.

Which of the following effects did the Treaty of Versailles have on the German economy during the 1920's? A.) Forced to pay war reparations to the Allies, Germany embarked on a massive program of job creation through public works. B.) Germany paid war reparations to the Allies by increasing taxes and reducing consumer spending. C.) Germany was forced to pay war reparations to the Allies using money it did not have, so the government printed more money, leading to hyper-inflation. D.) In order to pay its war reparations to the Allies, Germany had to rely on its colonies, leading to massive German emigration to Asia and Africa.

answer is C. The Treaty of Versailles forced Germany to pay reparations to the Allies for damages caused by the War. Germany attempted to pay the debt by seeking loans from the United States, and also by printing more money, which resulted in both efforts causing increasing amounts of inflation

Disputes involving all of the following contributed to the tensions that led to the War of 1812, EXCEPT: A. Free trade. B Territorial expansion. C. Abolition of slavery. D. Sailors' rights.

answer is C. The War of 1812 between the United States and Great Britain was caused primarily by disputes concerning America's right to engage in free trade with France, which country was then at war with England. In addition, the British claimed the right to stop and search U.S. Merchant Marine vessels and then impress into service and punish any deserters whom they found from the British Navy. The United States considered these sailors American citizens, and thus British actions to be illegal. In addition, Great Britain was supplying Indians along the Northwest frontier with the means to wage a continued war with American settlers in those areas. The question of slavery, on the other hand, was not a factor in the start of the War

In what state did the Revolutionary War begin? A. Maine. B. New York. C. Massachusetts. D. Virginia.

answer is C. The battle of Lexington and Concord were both on April 19th which started the Revolutionary War. From then forth, the Americans and British fought for control of the colonies

The Great Lakes region is most accurately characterized as: A.) A humid continental, warm summer climate zone. B.) A highland climate zone. C.) A humid continental, cool summer climate zone. D.) A semi-arid steppe climate zone.

answer is C. The continental climate zones commonly occur above 40º north latitude. In the summer the average temperature is above 50º F while in winter it is usually below freezing. The summer is shorter and cooler (except in cities like Chicago) in the Great Lakes region than further south in the Mid-Atlantic or Southern New England regions

Shawn is running for president. After a grueling campaign and election day, Shawn wins 26 states! Unfortunately, Shawn still loses. Why? A.) The states Shawn won were all in the South. B.) The states Shawn won were all in the north. C.)The states Shawn won did not have enough electoral votes to win. D.) Shawn did not win the popular vote.

answer is C. The members of the electoral college generally cast votes for the candidate elected by popular vote in a state. Each state has a number of votes based on the number of representatives and senators it has.. Members can be from either party. The larger states have more votes. In the 2000 election, Al Gore received more popular votes but G.W. Bush received more votes from the electoral college (after the Supreme Court determined the issue with Florida's votes.)

What name was given to the people from the North who traveled to the South during Reconstruction? A. Northern philanthropist. B. Carpetsharers. C. Carpetbaggers. D. Pursers.

answer is C. The nickname for these people came from the luggage which they would bring with them. The exterior was covered with old carpet to keep their belongings safe

Why were Julius and Ethel Rosenberg killed? A. For being mass murders. B. For being rapists. C. For being spies. D. For being Communist party members.

answer is C. The only two people who were ever executed for being spies in America, they were thought to have given information to the Soviet Union about nuclear weapons. Despite pleas for mercy, they were killed in Sing Sing prison

Cotton grows best in areas with a high amount of rainfall, but not too much, because cotton also needs a lot of direct sunlight. Also, from the time the cottonseeds are planted to when the bolls are plucked, there should not be any periods of frost or freezing temperatures. Which of the following regions of the United States meets these requirements? A. Northeast. B. Northwest. C. Southeast. D. Southwest.

answer is C. The region that best meets the requirements for growing cotton is the southeastern United States. The Northwest gets too much rain and not enough sunlight, while the Northeast is too cold for much of the year. In addition, before the introduction of cold-resistant varieties of cotton and, especially, advanced irrigation techniques, cotton was not very productively grown in the Southwest. The region with the best growing conditions for cotton is also the region historically most closely associated with slavery in the United States

What does demography measure? A. The amount of industry in a nation B. The number of farms in a nation C. The rate a population changes D. The types of industry a nation has

answer is C. The shift of the world's populations from agricultural to industrialized is a demographic transition. Demography is the study of rates of population change. It encompasses birth and death rates, migration patterns, and population shifts. When a nation shifts to an industrialized economy, the population moves from the country to the cities

The Act Prohibiting Importation of Slaves was a precursor to which of the following? A. 11th Amendment. B. 12th Amendment. C. 13th Amendment. D. 14th Amendment

answer is C. This Act, which stopped the slave trade in the United States, was paving the way for the 13th Amendment, which prohibited slavery altogether

In 1807, what was passed by Congress to prevent English ships from docking in US ports? A. British Act. B. Dock Act. C. Embargo Act. D. Stamp Act.

answer is C. This closed off all of the American ports to the British but did not cause much damage to British shipping. In the end, it was only American businessmen who were harmed. The act was repealed a few years later before the beginning of the war

The Pendleton Act of 1883 favored the use of: A. Political machines to gain votes. B. The patronage system to gain government jobs. C. The merit system to gain government jobs. D. Political parties to gain votes.

answer is C. This question tests your knowledge of government reform in the United States. The Pendleton Act of 1883 ruled that federal government jobs should be earned on the basis of qualifications, measured by tests, rather than political favoritism, based on party loyalty. It had no effect on the internal composition of political parties, or on local and state political "machines"

By 12,000 B.C.E., scholars believe that modern human beings had spread to all of the inhabitable continents, including Australia and South America. Around the globe, which of the following forms of food production did these early people have in common? A. Nomadic pastoralism. B. Rice farming. C. Hunting and gathering. D. Maize and bean farming

answer is C. This question tests your knowledge of the earliest forms of human societies. Hunting and gathering is the oldest form of human food production, and thus was common to all people everywhere in the world in 10,000 B.C.E. People did not begin farming rice until about 5000 B.C.E in China. Maize and bean farming began in Central and South America at around the same time. Nomadic pastoralism, or the herding of animals such as cows, sheep, and goats, only became widespread with the development of agriculture after 5000 B.C.E.

In May of 1768, a British warship entered the Boston Harbor, and in October of that same year, a regiment of British soldiers debarked on its shores. On which of the following historical events is the display of British military force most contingent? A. The Great Fire of London. B. The Boston Massacre. C. The Townshend Acts. D. The first voyage of Captain Cook.

answer is C. This question tests your knowledge of the escalating conflict between England and America in the years leading up to the Revolution. London burned a century earlier than the occupation under consideration, so event A is not the correct answer. Captain Cook served in the British Royal Navy during the French and Indian Wars, and then sporadically surveyed the coast of Newfoundland from 1763-1767. But his great voyage, that began in August, 1768, from Plymouth to the South Seas, bypassed Boston, so choice D is incorrect as well. The Massacre in B is relevant, but since it did not occur until March 5, 1770, the only correct answer is C: The Townshend Acts were a series of impositions proposed by Charles Townshend, Chancellor of the Exchequer. Goods that the colonists were forbidden to manufacture themselves, and obligated to buy only from England—tea, paper, paint, glass, lead—were levied heavier taxes. Colonial judges and governors, whose salaries, until that time, had been paid by colonial legislatures, were to be paid with the revenue of those taxes, and thus, by making their pay dependent on the Crown, their loyalty and judgments would be placed, in theory, beyond the colonists' control. Additional impositions established a new Crown-controlled American Customs Board that sought to enforce compliance with the Quartering Act. Resentment reigned throughout the colonies, and the unrest in Boston led the fearful Customs Board to request the presence of British troops, eventually leading to the Boston Massacre, and ultimately, to the Revolution.

All of the following events contributed to the United States' reversal of its official foreign policy of isolationism in April, 1917, EXCEPT: A. German submarines sank the passenger ship, the Lusitania. B. The British intercepted and decoded a German telegram suggesting that Mexico should invade the U.S. C. The Central Powers placed sanctions on American exports D. German submarines sank U.S. merchant vessels

answer is C. This question tests your knowledge of the events leading to United States' involvement in World War I. The Lusitania was sunk by German submarines in 1915. Despite the loss of American life, the United States insisted on maintaining its isolationist policy of neutrality. However, this event turned public opinion against Germany and eventually contributed to America's decision to go to war. A more direct cause of U.S. entry into the war was the Zimmerman Telegram, which offered military aid to Mexico if they attacked the U.S. Also, German submarines continued their attacks on U.S. merchant ships, further provoking America to war.

The Bill of Rights was added to the Constitution in order to: A.) Ensure that the national government was the sole sovereign power in the Union. B.) Promulgate the ideas of natural rights as defined in English common law. C.) Dispel fears that too much power would be given to the national government. D.) Place limits on the rights that individual citizens could legally claim

answer is C. This question tests your knowledge of the events surrounding the ratification of the Constitution. As the preamble to the Bill of Rights says, "The Conventions of a number of the States, having at the time of their adoption of the Constitution, expressed a desire to prevent misconstruction or abuse of its powers, that further declaratory and restrictive clauses should be added: And as extending the ground of public confidence in the government will best ensure the beneficent ends of its institution." It was "Resolved by the Senate and House of Representatives of the United States of America, in Congress assembled, two thirds of both Houses concurring, that the following Articles be proposed to the Legislatures of the several States, as amendments to the Constitution of the United States, all, or any of which Articles, when ratified by three- fourths of the said Legislatures, to be valid to all intents and purposes, as part of the said Constitution. These Articles, in addition to an Amendment to the Constitution of the United States of America, proposed by Congress, and ratified by the legislatures of the several States, pursuant to the fifth Article of the original Constitution." In other words, Massachusetts, New Hampshire, Virginia, New York, and North Carolina, fearing that the Constitution would grant possibly tyrannical powers to the central government, made the addition of a bill of rights a necessary condition of their ratification of the Document

In the second Book of the Histories, Herodotus reports that the priests of Memphis told him "that the Egypt to which the Hellenes come in ships is a land which has been won by the Egyptians as an addition, and that it isa gift of the river." Which of the following statements best accounts for Herodotus' report? A.) The ancestral gods of the Egyptians battled the Phrygians on the banks of the River and the goddess Nile gave them the land as a gift. B.) Silt deposited in the annual floods of the Nile was made into paste and applied to the skin to mark the different priestly ranks. C.) Silt deposited in the annual floods of the Nile made the land along the River's banks and marshes arable and inhabitable. D.) The Egyptians harnessed the power of the rapidly-flowing waters of the Nile River to drive mills to grind grain for flour and seeds for oil

answer is C. This question tests your knowledge of the influence of geography on the civilization of ancient Egypt. The cities of ancient (and modern) Egypt are built along a narrow strip of land on either side of the Nile River. Annual river floods deposit large quantities of silt or loess, which the inhabitants use as a substrate for the cultivation of the surplus of crops necessary to sustain the civilization. In addition, the waters of the Nile are employed for irrigation through the use of dikes and canals, as the arid region gets very little rain

Which of the following countries in Latin America has two capital cities? A.) Argentina. B.) Nicaragua. C.) Bolivia. D.) Costa Rica.

answer is C. This question tests your knowledge of the locations of cities of the world. Bolivia has two capital cities: Sucre, where the judicial branch and Supreme Court is located, and La Paz, where the legislative branch is located

A visitor to which of the following cities would find it useful to learn Portuguese? A. Lahore. B. La Paz. C. Luanda. D. Lulea.

answer is C. This question tests your knowledge of world geography in relation to language and the movements of human populations. Lahore is a city in Pakistan where Urdu, Punjabi, and Engish are spoken. La Paz is the capital of Bolivia, where Spanish is the main language, along with Aymara and Quechua. Lulea is a city in Sweden. Luanda is the capital of Angola, which, from the 16th Century until 1975, was an overseas territory of Portugal, which was a major player in the African slave trade. The official language of Angola is still Portuguese

What act helped set the guidelines as to where slavery was permitted in the United States? A. Texas Compromise. B. Act of Slavery. C. Missouri Compromise. D. Mexican War Act.

answer is C. This stated that any state which was above a certain line could be free while those below it could own slaves. Missouri was a slave state but later states such as Maine which was above the line was declared free

Champagne Charley, AKA Charles Townshend, the British Prime Minister in 1767, was angered by the American colonists resistance to British tax acts. What did he do to show his displeasure with this resistance? A. He raised the taxes on imports. B. He repealed the Stamp Act. C. He sent British troops to Boston. D. He censored American newspapers.

answer is C. To reinforce his resolve to "squeeze" some money out of the colonists, Townshend sent two regiments of British troops called "lobsterbacks" by the locals because of their red coats

Which of the following were the primary commanders of the armies fighting at the Battle of Yorktown in 1781? A. Benedict Arnold and Simon Fraser. B. Robert Howe and Archibald Campbell. C. George Washington and Charles Cornwallis. D. Nathanael Greene and Alexander Stewart.

answer is C. Washington commanded the American forces and Cornwallis, who eventually surrendered, commanded the British forces. The Battle of Yorktown was the last major battle of the American Revolution.

Which of the following correctly describes the intentions of Oliver North when he "pleaded the Fifth"? A. He refused to censor his speech. B. He refused to serve on a grand jury. C. He refused to be a witness against himself. D. He refused to cede property to the government

answer is C. When people "take [or plead] the Fifth," they are invoking a clause in the Fifth Amendment to the United States Constitution—"No person [...] shall be compelled in any criminal case to be a witness against himself"—and using this clause as the basis of their legal right to refuse to testify in court, since if they did, their words could be used as evidence leading to their possible indictment or conviction for a crime.

What famous general during the Civil War burned Atlanta? A. Lee. B. Grant. C. Sherman. D. Stonewall Jackson.

answer is C. While on campaign, he burned down the city so that the South would not be able to use it for their own purposes. He is said to be the advocate for modern warfare and is quoted as saying, "War is hell."

What Native American helped the Pilgrims last through their first winter in the New World? A. Sitting Bull. B. Pocahontas. C. Squanto. D. Sacajawea.

answer is C. Without his help, the Pilgrims would have died during the first winter. He brought them food and helped show them how they should prepare their crops for the next year

"The natural liberty of man is to be free from any superior power on Earth, and not to be under the will or legislative authority of man, but to have only the law of nature for his rule. The liberty of man, in society, is to be under no other legislative power, but that established, by consent, in the commonwealth; nor under the dominion of any will, or restraint of any law, but what that legislative shall enact, according to the trust put in it. Freedom then is not what Sir Robert Filmer tells us, Observations, A. 55. a liberty for every one to do what he lists, to live as he pleases, and not to be tied by any laws: but freedom of men under government is, to have a standing rule to live by, common to every one of that society, and made by the legislative power erected in it; a liberty to follow my own will in all things, where the rule prescribes not; and not to be subject to the inconstant, uncertain, unknown, arbitrary will of another man: as freedom of nature is, to be under no other restraint but the law of nature." The preceding passage comes from John Locke's Second Treatise of Government. In the commonwealth described by Locke: A.)Sovereign power is unitary and autocratic. B.) Sovereign power is plural and anarchic. C.) Sovereign power is unitary and bureaucratic. D.) Sovereign power is plural and democratic

answer is D. According to Locke, a person within the commonwealth is under the dominion of no one person's will in particular, but under the wills of all the citizens in consent with each other, and putting their trust in a legislative power that embodies that consent. Sovereign power in Locke's commonwealth is plural but not anarchic because the "standing rule" prevents each citizen from being "subject to the inconstant, uncertain, unknown, arbitrary will of another man." Sovereign power thus, is plural and democratic since it derives its authority from the consent of the citizens, and applies equally to them all: "The liberty of man in society, is to be under no other legislative power, but that established by consent, in the commonwealth; nor under the dominion of any will, or restraint of any law, but what that legislative shall enact, according to the trust put in it."

What nation was Alexander the Great born into? A. Greek. B. Persian. C. Hellenistic. D. Macedonian.

answer is D. Alexander the Great was born in Macedonia which was a separate area than the other nations in the region. Over time, he began to conquer them all and combined them into the Hellenistic Empire.

Which of the following is Alexander's MOST significant accomplishment in Greece? A. He introduced democracy. B. He created a space for intellectual discourse. C. He believed in peace. D. He unified the nation

answer is D. Alexander's most significant contribution to Greece is that he united the segregated city-states into a unified nation

What famous book was written by Adolf Hitler? A. According to Hitler. B. Fuhrer. C. Mein Putsch. D. Mein Kampf

answer is D. Also known as 'My Life', this book deals with Hitler's approach to his own life. In it, he exalts the aryan race as being the main race of the world.

All of the following occurred during Mao Zedong's Cultural Revolution, EXCEPT: A. The universities were shut down. B. Dissidents were imprisoned. C. Elites were re-educated. D. Military power increased.

answer is D. Although China's military power had been greatly expanding since the Communist Revolution in 1949—even to the point of testing an atomic bomb in 1964, one of the effects of the Cultural Revolution of the late 1960s was a drastic decrease in military power, as well as economic productivity

For what nationality was Columbus sailing under when he discovered the West Indies? A. England. B. Italy. C. Portugal. D.Spain.

answer is D. Although Columbus himself was Italian, the Italian royalty did not have enough money to sponsor his voyage. Spain did though and with their sponsorship of Columbus they managed to gain a solid footing in the New World

Which of the following statements is FALSE regarding anthropological fieldwork? A. Simple observations are made about the target population. B. Data is collected about the target population. C. Various ideas about the target population are tested. D. Therapy is provided to the target population.

answer is D. Although anthropologists sometimes participate in the targeted culture, gathering information and testing ideas are the main purpose of fieldwork. Therapy is not provided by anthropologists.

What British inventor created the forerunner to the modern computer? A. Baggage. B. Pascal. C. Voltaire. D. Babbage.

answer is D. Although he never completed his machine, successors were based upon his designs. This was later seen as being the first computer

What traitor of the Revolutionary War was hanged for giving away secrets to the British? A. Nathan Hale. B. Ethel Rosenberg. C. Benedict Arnold. D. none of the above

answer is D. Benedict Arnold was a traitor of the United States to the British but escaped hanging. Instead, he became a British general and settled in London after the war. The United States spy Nathan Hale was hanged by the British for giving British secrets to Americans

Which of the following BEST illustrates how Carl Rogers would approach personality? A.) As a major focus on gender schema and sex roles. B.) Understanding the significance of biological factors and unconscious forces. C.) Revisiting childhood experiences to determine the source of neuroses. D.) Emphasis on personal growth and self-actualization

answer is D. Carl Rogers was a humanistic psychologist who believed that all individuals possess a large capacity for self-direction and the ability to understand and direct their own development

On the size and spacing of human settlements: 1.The larger the size of the settlements, the fewer in number they will be. 2.The larger the growth of the settlements, the greater the distance between them. 3. As a settlement increases in size, the range and number of its functions will increase . 4. As a settlement increases in size, the number of higher-order services will also increase. The four points above detail the main ideas behind which of the following? A.) Density thresholds. B.) Trans-cultural diffusion. C.) Scarcity. D.) Central Place Theory.

answer is D. Central Place Theory suggests that centralization is a natural principle of order, and that human settlements follow it. It also argues that there are laws determining the number, size and distribution of towns

Which of the following is NOT one of the main theoretical approaches to sociology? A. Social conflict. B. Feminism. C. Symbolic interactionism. D. Cognitivism.

answer is D. Cognitivism is a paradigm from the field of psychology, not sociology. Symbolic interactionism (the self is a purposeful and creative social product), feminism (political, social, and economic equality of women) and social conflict theory (groups with more power exploit those with less power) are all sociological theories

Your national culture has been living alongside another for generations. Your national boundaries are being redrawn based on this cultural difference. What type of boundaries are being drawn? A.) Artificial B.) Natural C.) Superimposed D.) Consequent

answer is D. Consequent boundaries are also called ethnographic. They respond to cultural influences. Superimposed boundaries are created by an outside force which disregards previous cultural borders. A natural boundary is formed by a land form or body of water. Rivers, mountains, lakes or valleys might be natural boundaries. An artificial boundary is a surveyed boundary using lines of latitude and longitude

Which city does not have a US Mint? A. Denver. B. Philadelphia. C. San Francisco. D. Boston.

answer is D. Denver and Philadelphia create the coins which are being used throughout the nation. Special proof coins are created in the San Francisco area

During the 16th century, called the Elizabethan age, the gentry ate heartily. But many of them had rotten teeth. Why was this so? A. They ate too many sweets. B. They didn't eat many vegetables. C. They drank too much wine. D. All of the choices are correct.

answer is D. During the Elizabethan age, the gentry ate well but had rotten teeth. They ate much meat and fish and very few vegetables. They drank a lot of wine and ate sweets. The experts advised them to brush their teeth with sugar. Even the queen had rotted and painful teeth.

After the Congress passed the final draft of the Constitution, they had to get the states to ratify this. How does a state ratify such a document? A. They call for a general election. B.They elect delegates to consider it. C.The governor ratifies it. D. The governor appoints delegates to consider it.

answer is D. Each state elected delegates who considered the proposed Constitution. Nine states needed to approve it so that it could become the law of the land

What is the difference in the representation in the House and Senate? A. The Senate has one member per state and the House has two. B. The Senate has two members per state and the House has one. C. The House has an equal number of delegates from each state and the Senate's membership is based on population. D. The Senate has two representatives from each state. The House has representation based on a state's population.

answer is D. Each state has equal representation in the Senate with two Senators. The number of Representatives from each state in the House is based on population. This was the compromise agreed to by the big and small states at the Constitutional Convention.

The political activities of which of the following women was instrumental in the passing of the 19th Amendment to the Constitution? A. Elizabeth Cady Stanton. B. Susan B. Anthony. C. Lucretia Mott. D. Alice Paul.

answer is D. From 1916 until 1920, Alice Paul and other members of the National Woman's Party used peaceful demonstrations, civil disobedience, and other tactics to exert pressure on politicians to pass the 19th Amendment, which states, "The right of citizens of the United States to vote shall not be denied or abridged by the United States or by any State on account of sex. Congress shall have power to enforce this Article by appropriate legislation."

The American Federation of Labor was founded by: A. John Mitchell. B. Terence V. Powderly. C. Mary Harris Jones. D. Samuel Gompers.

answer is D. Gompers founded the American Federation of Labor in 1886, with aims to secure skilled workers shorter hours, safer working conditions, and higher wages through the process of collective bargaining.

General Robert E. Lee surrendered to Ulysses S. Grant at: A. Vicksburg. B. Chancellorsville. C. Atlanta. D. Appomattox.

answer is D. Grant's and Lee's units had skirmished over the course of several days in early April, 1865. But Lee was eventually cut off from expected sources of supply by superior Union forces, including a cavalry unit led by George Armstong Custer. Lee therefore, chose the only logical choice given his limited options: Surrender or continue to face starvation and decimation. He chose the former on April 10. The entire Confederate force then laid down their arms two days later

What religion was created by Henry VIII? A. Catholicism. B. Huguenots. C. Protestantism. D. Anglican.

answer is D. He created the church so that he would be able to get a divorce from his current wife and marry another. His daughter, Elizabeth I continued on the religion which exists as the official religion of royalty, to this day

Who was the first person to reach the top of Mount Everest? A.) Perry. B.) Smith. C.) Houston. D.) Hillary.

answer is D. Hillary was the first person to make it to the top, assisted with sherpas. Later on, he was knighted and returned to Nepal to help the people in the region.

Leif Eriksson and his brother, Thorvald, were among the first from this area to travel to North America. What area did they hail from? A. France B. Scotland C. England D. Scandinavia

answer is D. Historians believe that between the 9th and 10th centuries, the Vikings traveled from Scandinavia to North America. Viking means "sea raider" or "pirate" but most were farmers or herders. They were the first Europeans to travel to North America. Leif and Thorvald were Vikings. Thorvald was the first white man killed by the Indians

Throughout history, when have empires been most vulnerable to invasion and decline? A. When they engaged in naval battles. B. When they traded with their neighbors. C. When they were most compact. D. When they were most spread out.

answer is D. History seems to indicate that many empires, such as the ancient Egyptian, Mesopotamian, Greek, and Roman Empires, have been at their most vulnerable when just past the peak of their prime—material wealth was greatest because so many goods were flowing in from trade and conquest alike; military power was greatest because so many soldiers were required to patrol the extensive borders of an expansive territory. Envious and resentful of these displays of wealth and power, the empire's enemies could take advantage of cracks in the territorial armor, of sparsely manned outposts, of fissures in the belligerent buffers along the frontier, in order to eat away at the empire gradually from outside, to infiltrate, invade, and eventually destroy it

Which of the following leaders was NOT a dictator? A. Hitler. B Mussolini. C. Stalin. D. Gandhi.

answer is D. Hitler, Mussolini and Stalin were all dictators. Gandhi was a leader who advocated for change through mass civil disobedience

Why did the Spanish plantation owners in the western hemisphere in the 16th century import slaves from Africa? A.) The Indians were generally nomadic hunters and gatherers. B.) The Indian population had increased and they were afraid of revolt. C.) The Africans were better workers. D.) The Indian population had diminished.

answer is D. Illnesses like measles and smallpox killed many of the Indians. They feared the cruelty of the conquistadors and fled. So the plantation owners imported slaves from Africa. They needed workers to grow sugarcane or tobacco, both native to America

The U.S. is a government of "laws" not "men." This principle of the U.S. that even the President must obey the law is based on what earlier document? A. The Alien and Sedition Acts B. The Articles of Confederation C. Plato's Republic D. The Magna Carta

answer is D. In 1215, England's powerful barons forced King John to sign a document which limited the power of the king to tax them and protected the rights of landowners. The Magna Carta helped establish the principle that no one is above the law. It also argued that individuals had certain rights granted by their humanity

In 1492, Columbus sailed from Spain across the Atlantic to the New World. What other event began in Spain around the same time? A.) The Spanish Armada B.) The Spanish Flu C.) The Spanish Conquest D.) The Spanish Inquisition

answer is D. In 1492, the Jews were expelled from Spain. Queen Isabella, a Catholic, assembled a religious court called "The Inquisition." It forced non-Catholics to convert or leave Spain. If they refused, they were tortured or executed. Some people objected but were ignored

In 1517, the Protestant Reformation started in Germany. Who started this movement? A. Martin Buber B. John Calvin C. Jakob Ammon D. Martin Luther

answer is D. In 1517, Martin Luther, a professor and priest, published his list of demands for reform in the Roman Catholic Church. He nailed his 95 Theses to his church door in Wittenberg, Germany. He began the movement called the Protestant Reformation

During the Middle Ages, feudalism was an economic and social system binding rulers, landowners, farmers, and warriors into codified relationships of vassalage, patronage, and labor in which of the following regions of the world? A. New Spain and New Guinea. B. Polynesia and Mali. C. China and North America. D. Japan and Europe.

answer is D. In Japan and Europe c. 700-1300 C.E., the land was divided into manors or fiefs controlled by vassals—knights in Europe, samurai in Japan. On each manor, vassals granted serfs the right to work the fields and make things, in exchange for their loyalty to the manor, as well as the largest share of their produce. Going up the ladder, the vassals pledged their allegiance to the nobility, who pledged theirs to the highest rulers. It was in exchange for such a distribution of loyalty and labor that the crown dispersed its lands, with all engaged in the mutual defense of the territory thus loosely united by bonds of faith and feudal "finance"

The lowdown on baguettes in Phildelphia is this: There are not many bakeries that make them, they vary in price from $1.50 to $4.00, and the bakeries that do sell them, sell out pretty quickly. Paris, by contrast, is a baguette lover's heaven. There is no end to baguettes in Paris. There are bakeries on every street corner, and they only cost from $1.00 to $1.50; and best of all, there is also no end to baguette lovers. Which of the following statements most accurately describes the economics of the baguette market in Philadelphia in comparison to Paris? A.) The baguette market in Philadelphia seems to be an oligopoly with collusion, while the market in Paris is a non-colluding oligopoly with price ceilings. B.) The baguette market in Philadelphia seems to be in a state of immature perfect competition, while the market in Paris is a subsidized oligopoly. C.) The baguette market in Philadelphia seems to be in a state of mature perfect competition, while the market in Paris is a natural monopoly subsidized by the government. D.) The baguette market in Philadelphia seems to be in a state of monopolistic competition, while the market in Paris is in a state of mature perfect competition.

answer is D. In both Philadelphia and Paris there seem to be high demand for baguettes. So the greater variation in prices in Philadelphia, coupled with the low numbers of bakeries, would seem to indicate a state of monopolistic competition. If there were high demand, few bakeries, and a smaller range of prices, immature perfect competition would be indicated. But this is not the case. In Paris, the lower, and more consistent, prices coupled with high demand as well as many bakeries with high output seem to indicate a state of mature perfect competition in which the number of firms entering or exiting the market has been stabilized

Which of the following nations has the highest population of Muslims? A. Saudi Arabia. B. Pakistan. C. Iran. D. Indonesia

answer is D. Indonesia has an estimated population of 238 million, of which 86% are Muslim. The population of Saudi Arabia is 28 million, Pakistan is 166 million, and Iran about 71 million

"But I take higher ground. I hold that in the present state of civilization, where two races of different origin and distinguished by color, and other physical differences, as well as intellectual, are brought together, the relation now existing in the slave-holding states between the two is, instead of an evil, a good—a positive good." The preceding sentiments were held by which of the following American politicians? A. John Quincy Adams. B. Daniel Webster. C. Andrew Jackson. D. John Caldwell Calhoun.

answer is D. John C. Calhoun of South Carolina served as Vice-President under first, John Quincy Adams, then Andrew Jackson. He resigned from the office of Vice-President during the Nullification Crisis, and ran for the Senate. He was elected to the Senate on December 28, 1832, and five years later he delivered a speech in the Senate justifying his views on slavery, going well beyond the position even a slave-owning politician such as Jackson would have stated in public

The United States system of laws regulates a conflict between rights and responsibilities. We have a right to freedom, but a responsibility to act morally and for the common good. Which early American group first established a charter regarding this conflict? A. The Pilgrims B. The Quakers C. The Mennonites D. The Puritans

answer is D. John Winthrop lead a group of Puritans to Massachusetts Bay in 1630. The charter they used has had a big impact on roots of our modern corporate system, representative form of government, the American legal system and the moral conflict between rights and responsibilities. Citizens have a right to think and act freely but are constrained by the rights of others. The Puritans were hard on those who disagreed with their religion, such as the Quakers who also lived there. They were often banished or beaten.

Who was the first ruler of the Holy Roman Empire? A. Charles Martel. B. Richard the Lionhearted. C. Francis II. D. Charlemagne.

answer is D. On Christmas Day 800 C.E., Charlemagne was crowned by Leo III. This empire lasted until Francis II was defeated by Napoleon

Which of the following BEST describes why Adler considered Freud's concept of sublimation to be a "healthy defense mechanism"? A. Because sublimation differentiates kind people from unkind people. B. Because sublimation is the result of a highly-educated mind. C. Because sublimation is both harmless and pleasant. D. Because sublimation ultimately benefits humanity

answer is D. Originally, Freud introduced sublimation as the redirection of inappropriate sexual and violent tendencies into more socially acceptable behaviors. Adler said that sublimation was a "healthy defense mechanism" because its outcome is behavior that benefits the individual and society.

Which of the following is considered one of the most important methods of data collection for anthropologists? A.Quasi-empirical method. B. Field experiment. C. Quantitative method. D. Participant-observation.

answer is D. Participant-observation is long-term, experiential immersion into the area of research, and is a significant method of study for anthropologists

The continents of the world are not stable masses. They drift on tectonic plates that move as much as several inches a year. When they collide an earthquake might occur. What is the spot on the surface right above the focus of an earthquake called? A. The focal point B. The midpoint C. The Tectonic center D. The epicenter

answer is D. Right above the focal point of an earthquake is the epicenter. An earthquake's damage is calculated by the amount of destructiveness which is caused to life and property above it

When the Articles of Confederation were submitted in the summer of 1777, how many states were involved in their ratification? A. 10 States. B. 15 States. C. 12 States. D. 13 States.

answer is D. The 13 States, which before that time were considered colonies, were New Hampshire, Massachusetts, New York, New Jersey, Delaware, Maryland, Rhode Island, Connecticut, Pennsylvania, Virginia, North Carolina, South Carolina, and Georgia

Which group settled in the colony of New York for the purposes of maritime trade? A. The French B. The English C. The Spanish D. The Dutch

answer is D. The Dutch were the most successful maritime traders in the Old World. They set up a colony in the New World called New Amsterdam at the mouth of the Hudson River. It was founded for the purpose of trade run for the benefit of the Dutch West Indies stockholders. In 1664, the British came to attack the settlement, but they surrendered. The settlement was renamed New York after James, The Duke of York. Some of the colony was given to James's friends and this became New Jersey

Why do cartographers use elevation and relief on a map? A.To show the oceans and landforms B. To determine distance C. To show the tilt of the Earth D. To show the earth's irregularities, such as rivers and valleys

answer is D. The Earth's surface is irregular. Cartographers use elevation and relief to represent these irregularities. The baseline is often above or below sea level. Cartographers use shading or hatching to represent elevation and relief

Roger Williams of Providence, R.I. was ejected from the Massachusetts Bay Colony for a belief the U.S. guarantees today. What was it? A.Self-government B. Limiting the rights of a leader C. Educating children D. Religious Freedom

answer is D. The Mayflower Compact adopted by the Pilgrims in 1620 stated that political authority flows from the people, not from the king. The Massachusetts School Laws of 1642 required parents to educate every child. Roger Williams Letter to Providence, 1655, argued for religious freedom. The Magna Carta of 1215 limited the powers of the king

The Marshall Plan had all of the following effects, EXCEPT: A.) Facilitating the economic growth of Western Europe. B.) Increasing American cultural influence in Europe. C.) Laying the foundation for European integration. D.) Dividing Berlin into eastern and western halves

answer is D. The Potsdam Agreement of 1945 divided Germany and Berlin into French, British, Russian, and American zones. The split between West Berlin (American, French, and British sectors) and Russian-controlled East Berlin was strenghtened during the Berlin Blockade of 1948-1949. By the terms of the Marshall Plan, trade barriers were eliminated among the countries of western Europe, thus laying the foundation for the integration that would become the European Union

What group of Greek city-states was led by Athens and had a central base at Delos? A. Athenian League. B. Aegean League. C. Greek Super League. D. Delian League.

answer is D. The city-states came together in order to remain stronger than If they were apart because of the Persian threat. The Battle of Chaeronea eventually crushed the league and caused it to be dissolved

Which of the following stages of Jean Piaget's theory begins at around 12 years of age? A. Concrete operations. B. Sensorimotor. C. Preoperational. D. Formal operational.

answer is D. The formal operational stage, which is the final stage in Piaget's theory, begins around the onset of puberty and continues into adulthood.

In general, the littoral zone can be defined as which of the following? A.) Tropical zone between humid mangrove ecosystem and inland hardwood forest. B.) Mountain zone between timberline and tundra. C.) Arid zone between grasslands and desert. D.) Coastal zone between high tide water mark and submerged areas just offshore

answer is D. The littoral zone is an ecosystem of coastal regions. It is generally defined as the area between the water mark at highest tide, or perhaps slightly further inland, and submerged areas offshore that receive plenty of sunlight

For the purposes of representation in the House of Representatives, the populations within Congressional districts are assessed: A. Every two years. B. Every three years. C. Every six years. D. Every ten years.

answer is D. The primary function of the census taken every ten years in the United States is to determine the allotment of seats in the House. As Article I, Section 2 of the Constitution states: "Representatives and direct taxes shall be apportioned among the several States which may be included within this Union, according to their respective numbers, which shall be determined by adding to the whole number of free persons, including those bound to service for a term of years, and excluding Indians not taxed, three fifths of all other persons. The actual enumeration shall be made within three years after the first meeting of the Congress of the United States, and within every subsequent term of ten years, in such manner as they shall by law direct."

"Above all, we want equal political rights, because without them our disabilities will be permanent. I know this sounds revolutionary to the whites in this country, because the majority of voters will be Africans. This makes the white man fear democracy. But this fear cannot be allowed to stand in the way of the only solution which will guarantee racial harmony and freedom for all. It is not true that the enfranchisement of all will result in racial domination. Political division, based on colour, is entirely artificial and, when it disappears, so will the domination of one colour group by another. The ANC has spent half a century fighting against racialism. When it triumphs it will not change that policy. [...] During my lifetime I have dedicated myself to this struggle of the African people. I have fought against white domination, and I have fought against black domination. I have cherished the ideal of a democratic and free society in which all persons live together in harmony and with equal opportunities. It is an ideal which I hope to live for and to achieve. But if needs be, it is an ideal for which I am prepared to die." Which of the following political leaders is the author of the preceding words? A. Malcolm X. B. Martin Luther King, Jr. C. Kwame Nkrumah. D. Nelson Mandela.

answer is D. The quoted passage was made by anti-apartheid activist Nelson Mandela at the opening of his defense at the Rivonia trial in the Pretoria Supreme Court on April 20, 1964. Mandela, who had helped found Umkhonto we Sizwe, or "Spear of the Nation"—the military wing of the African National Congress (ANC)—on December 16, 1961, was eventually found guilty of sabotage and was sentenced to life imprisonment.

Colonial soldiers during the Revolutionary War were known as what? A. Hourlings. B. Colony Soldiers. C. Instatroops. D. Minutemen.

answer is D. These troops were said to be ready at a minute's notice which is where they got their name from. Most of them were farmers or other laborers and did not have professional army training

What two Greek city-states fought against each other in the Peloponnesian War? A. Corinth-Sparta. B. Athens-Ithaca. C. Athens-Corinth. D. Athens-Sparta.

answer is D. These two city-states shared different ideals about what should be pursued by their citizens. When Sparta felt threatened by the imperialistic approach of Athens, it launched a war against that city-state. Athens surrendered and Sparta installed tyrants to rule the city-state

Which of the following rivers is NOT in Africa? A. The Congo River. B. The NigerRiver. C. The Nile River. D. The Salween River.

answer is D. This question tests your knowledge of the locations of some major rivers in the world. The Congo, the second largest river in Africa, forms a great south-northwest arc bisecting the Democratic Republic of the Congo, gently loops back to the southwest to form the curving boundary of that country, and the Republic of the Congo lining the river's northwest banks, before emptying into the Atlantic Ocean. The world's longest river, the Nile, flows north from central Africa, across the north African nation of Egypt, and empties into the Mediterranean. The Niger River, the third longest in Africa, flows in a great northeast-southern jagged arc from the Guinea Highland through Nigeria to the Atlantic. The Salween River starts in Tibet, flows south through China, Burma, and Thailand, and empties into the Andaman Sea

What revolution helped bring Louis-Philippe to power in France during 1830? A. June Revolution. B. French Revolution. C. Peasants' Revolution. D. July Revolution.

answer is D. This was done over a period of three days in which Charles X abdicated from the throne. The new ruler was said to be the Citizens' King.

Thomas Paine's pamphlet "Common Sense" had a big influence on early America. What arguments did it present? A. An end to slavery and establishing a place for freed slaves to live B.Women's rights and animal's rights C. An end to the monarchy and passive resistance to the army D. An end to the monarchy and the right to self-government

answer is D. Tom Paine was an early advocate of women's and animal's rights. He also was an anti-slavery supporter. Common Sense dealt with a call for the end of the monarchy and the people's right to govern itself. He was instrumental in converting thousands of Americans to the cause of liberty

Patrick Henry in 1775 argued for a right which is very controversial in modern America. What was it? A.) The right to an abortion B.) The right for religions to build houses of worship in a city or town C.) Against discrimination based on gender or sexual orientation D.) The right to bear arms

answer is D. When Patrick Henry spoke his famous line to the Virginia Convention in 1775, "Give me liberty or give me death" he was arguing for the right to form a citizen's militia. This right has been the subject of many discussions about just what the "right to bear arms" includes

Ptolemy, a Greek geographer who lived in Egypt, made a map which mislead both Columbus and Magellan. What misconception cost Magellan many months of extra travel? A. Ptolemy's map had misleading lines of latitude. B. Ptolemy thought that North America was smaller than it was. C. Magellan thought that the Pacific Ocean would be calm and much smaller. D. Magellan thought that you could sail between North and South America.

Answer: C Magellan thought the Pacific Ocean would be calm and much smaller. Magellan had studied Ptolemy's maps. In 1519, Magellan sailed west from Spain. He found a passageway, later named the Magellan Strait near the tip of South America which led to the Pacific. He sailed across the Pacific. He had thought the Pacific would be a calm ocean (Pacific=peaceful.) Along the way, he met cannibals, stayed with people he named the Patagones and took two of them with him to display in Spain. When he realized that the Pacific was vast and wild, he continued . He managed to miss every island between South America and Guam and landed in the Spice Islands having survived storms, near mutinies and starvation. They landed in the Philippine Islands. He was killed in a fight there. After three years of adventures, his last ship returned to Spain. He and his shipmates wrote about his adventures which is why we know about them today. (Actually a Moluccan slave named Enrique who accompanied Magellan had been enslaved by Spanish traders and taken to Spain. He was the first person to encircle the globe but Magellan gets the credit.)


Kaugnay na mga set ng pag-aaral

✓NBCOT Questions #11: 551-610 (7/15/19)

View Set

Chapter 5 HW - Financial Management

View Set

Ch 6 Consumer Purchasing Strategies and Wise Buying of Motor Vehicles

View Set

Practice Accounting Chapter 5 &6

View Set

Fundamental of nursing Ch3: Health, illness, and disparities

View Set

EBIO Midterm #2 Homework Questions

View Set

Financial Management Ch. 11 Risk and Return

View Set

Chemistry: Lab Skills, Lab quiz: Flame test

View Set

MGT304 Chapter 1: What is Organizational Behavior

View Set